[obm-l] Re: [obm-l] Re: [obm-l] Re: [obm-l] Desigualdade com potências

2018-04-29 Por tôpico Douglas Oliveira de Lima
Valeu Ralph, thanks.

Douglas Oliveira.

Em dom, 29 de abr de 2018 16:49, Ralph Teixeira <ralp...@gmail.com>
escreveu:

> Que tal assim:
>
> POR BAIXO (BEM folgado): Como 3^3=27<32=2^5, temos
> 3^100<(3^3)^34<(2^5)^34=2^170. Portanto
> 3^100+2^100<2^170+2^100<2^170+2^170=2^171<2^200=4^100.
> POR CIMA (mais apertado!): Como 3^7=2187>2^11=2048, temos
> 3^100=9.(3^98)>9.(2^154)>(2^3).(2^154)=2^157. Somando 2^100, ficamos abaixo
> de 2.(2^157)=2^158=4^79.
>
> Abraco, Ralph.
>
> 2018-04-29 13:09 GMT-03:00 Anderson Torres <torres.anderson...@gmail.com>:
>
>> 2018-04-29 8:45 GMT-03:00 Douglas Oliveira de Lima
>> <profdouglaso.del...@gmail.com>:
>> > Prove que 4^79<2^100+3^100<4^100, usando matemática elementar.
>> >
>>
>> O desejo de trapacear isso com log é muito forte :)
>>
>> Isso equivale a mostrar que
>>
>> 2^158-2^100<3^100<2^200-2^100
>>
>> Ou
>>
>> (2^58-1)*2^100<3^100<(2^100-1)*2^100
>>
>> Ou talvez
>>
>> 2^58 < (3/2)^100+1 < 2^100
>>
>> Daqui, tenho poucas ideias para a desigualdade mais à esquerda...
>>
>> > Douglas Oliveira.
>> >
>> > --
>> > Esta mensagem foi verificada pelo sistema de antivírus e
>> > acredita-se estar livre de perigo.
>>
>> --
>> Esta mensagem foi verificada pelo sistema de antivírus e
>>  acredita-se estar livre de perigo.
>>
>>
>> =
>> Instruções para entrar na lista, sair da lista e usar a lista em
>> http://www.mat.puc-rio.br/~obmlistas/obm-l.html
>> =
>>
>
>
> --
> Esta mensagem foi verificada pelo sistema de antivírus e
> acredita-se estar livre de perigo.

-- 
Esta mensagem foi verificada pelo sistema de antiv�rus e
 acredita-se estar livre de perigo.



[obm-l] Desigualdade com potências

2018-04-29 Por tôpico Douglas Oliveira de Lima
Prove que 4^79<2^100+3^100<4^100, usando matemática elementar.

Douglas Oliveira.

-- 
Esta mensagem foi verificada pelo sistema de antiv�rus e
 acredita-se estar livre de perigo.



Re: [obm-l] Desigualdade

2018-04-17 Por tôpico Douglas Oliveira de Lima
Nao entendi esse a_k Produto.

por exemplo se fossem a_1, a_2 e a_3, entao seria
1/a_1[(a_3)^2-(a_1)^2][(a_2)^2-(a_1)^2]
+1/a_2[(a_3)^2-(a_2)^2][(a_1)^2-(a_2)^2]+1/a_3[(a_2)^2-(a_3)^2][(a_1)^2-(a_3)^2],
é maior que zero , é isso?

Douglas Oliveira.

Em ter, 17 de abr de 2018 00:49, Artur Costa Steiner <
artur.costa.stei...@gmail.com> escreveu:

> Sejam a_1, a_n  números positivos, distintos dois a dois, e, para k =
> 1, ... n, definamos
>
> p_k = a_k Produto (j = 1, n, j <> k) ((a_j)^2) - (a_k)^2)
>
> Mostre que 1/p_1 ... + 1/p_n > 0
>
> Artur
>
>
>
> --
> Esta mensagem foi verificada pelo sistema de antivírus e
>  acredita-se estar livre de perigo.
>
>
> =
> Instruções para entrar na lista, sair da lista e usar a lista em
> http://www.mat.puc-rio.br/~obmlistas/obm-l.html
> =
>

-- 
Esta mensagem foi verificada pelo sistema de antiv�rus e
 acredita-se estar livre de perigo.



Re: [obm-l] Soma (k = 1, n) 1/P'(r_k) = 0

2018-04-16 Por tôpico Douglas Oliveira de Lima
Entao a questao é até que ponto ela é verdadeira , pois funciona para casos
elementares.

Douglas Oliveira

Em dom, 15 de abr de 2018 22:29, Bernardo Freitas Paulo da Costa <
bernardo...@gmail.com> escreveu:

> 2018-04-15 13:09 GMT-03:00 Douglas Oliveira de Lima
> <profdouglaso.del...@gmail.com>:
> > Usa o polinomio de Lagrange , nao é nada obvia mesmo.
>
> Como usa Lagrange, a fórmula segue para k = 0, 1, ... n-1
> (interpolando em n pontos, vamos até grau n-1).  E é, de fato, falso
> para k = n, use P(x) = (x-1)(x+1).
>
> Além disso, mesmo para k = n-1, a demonstração por complexa não se
> aplica mais (o grau dá errado...), e o mesmo polinômio serve para
> mostrar que a soma não dá mais zero.
>
> Abraços,
> --
> Bernardo Freitas Paulo da Costa
>
> --
> Esta mensagem foi verificada pelo sistema de antivírus e
>  acredita-se estar livre de perigo.
>
>
> =
> Instru�ões para entrar na lista, sair da lista e usar a lista em
> http://www.mat.puc-rio.br/~obmlistas/obm-l.html
> =
>

-- 
Esta mensagem foi verificada pelo sistema de antiv�rus e
 acredita-se estar livre de perigo.



Re: [obm-l] Soma (k = 1, n) 1/P'(r_k) = 0

2018-04-15 Por tôpico Douglas Oliveira de Lima
Usa o polinomio de Lagrange , nao é nada obvia mesmo.

Douglas Oliveira.

Em sex, 13 de abr de 2018 13:41, Claudio Buffara <claudio.buff...@gmail.com>
escreveu:

> Essa identidade:
>  x^k=soma (i=1,...,n)(x_i)^k.P(x)/(x-x_i).P'(x_i)
> não me parece nada óbvia.
>
> []s,
> Claudio.
>
>
> 2018-04-13 5:56 GMT-03:00 Douglas Oliveira de Lima <
> profdouglaso.del...@gmail.com>:
>
>> Entao, sendo x^k=soma (i=1,...,n)(x_i)^k.P(x)/(x-x_i).P'(x_i) , é só
>> igualar os coeficientes de x^(n-1) e pronto, a identidade se torna ate mais
>> genérica
>>
>> Soma (i= 1, n) (x_i)^k/P'(x_i) = 0
>>
>> Obs: x_i sao raizes.
>>
>> Abraco
>>
>> Douglas Oliveira.
>>
>>
>>
>>
>> Em 8 de abr de 2018 20:50, "Artur Steiner" <artur.costa.stei...@gmail.com>
>> escreveu:
>>
>> Seja P um polinômio complexo, de grau n >= 2, que tenha n raízes simples
>> r_1, ... r_n. Mostre que Soma (k = 1, n) 1/P'(r_k) = 0.
>>
>> Para quem conhece um pouco de análise complexa, isto é corolário de um
>> resultado geral. Mas parece que pode ser provado sem análise complexa.
>>
>> Artur Costa Steiner
>>
>> --
>> Esta mensagem foi verificada pelo sistema de antivírus e
>> acredita-se estar livre de perigo.
>>
>>
>>
>> --
>> Esta mensagem foi verificada pelo sistema de antivírus e
>> acredita-se estar livre de perigo.
>>
>
>
> --
> Esta mensagem foi verificada pelo sistema de antivírus e
> acredita-se estar livre de perigo.

-- 
Esta mensagem foi verificada pelo sistema de antiv�rus e
 acredita-se estar livre de perigo.



[obm-l] Re: [obm-l] Re: [obm-l] Combinatória

2018-04-14 Por tôpico Douglas Oliveira de Lima
Entao , veio de quantas soluções inteiras positivas existem para x+yz+w=100.

Douglas Oliveira.

Em sáb, 14 de abr de 2018 13:37, Claudio Buffara <claudio.buff...@gmail.com>
escreveu:

> Que eu saiba, só no braço, mesmo...
>
> n(k) é uma fórmula envolvendo os expoentes da decomposição de k em fatores
> primos.
> Não conheço nenhuma expressão de n(k) em função de k diretamente.
>
> De onde veio este problema?
>
> []s,
> Claudio.
>
>
> 2018-04-10 18:11 GMT-03:00 Douglas Oliveira de Lima <
> profdouglaso.del...@gmail.com>:
>
>> Caros amigos , retomando o raciocinio, rs, estou com um problema um tanto
>> interessante que nao sei como fazer:
>>
>> Existe algum jeito de calcular o valor do somatório dos produtos
>> n(k).(101-k) onde k varia de 1 a 98 e n(k) é o número de divisores de k.
>>
>>
>> Qualquer ajuda será bem vinda.
>>
>>
>> Abraco do
>> Douglas Oliveira.
>>
>>
>> --
>> Esta mensagem foi verificada pelo sistema de antivírus e
>> acredita-se estar livre de perigo.
>
>
>
> --
> Esta mensagem foi verificada pelo sistema de antivírus e
> acredita-se estar livre de perigo.

-- 
Esta mensagem foi verificada pelo sistema de antiv�rus e
 acredita-se estar livre de perigo.



Re: [obm-l] Soma (k = 1, n) 1/P'(r_k) = 0

2018-04-13 Por tôpico Douglas Oliveira de Lima
Entao, sendo x^k=soma (i=1,...,n)(x_i)^k.P(x)/(x-x_i).P'(x_i) , é só
igualar os coeficientes de x^(n-1) e pronto, a identidade se torna ate mais
genérica

Soma (i= 1, n) (x_i)^k/P'(x_i) = 0

Obs: x_i sao raizes.

Abraco

Douglas Oliveira.




Em 8 de abr de 2018 20:50, "Artur Steiner" 
escreveu:

Seja P um polinômio complexo, de grau n >= 2, que tenha n raízes simples
r_1, ... r_n. Mostre que Soma (k = 1, n) 1/P'(r_k) = 0.

Para quem conhece um pouco de análise complexa, isto é corolário de um
resultado geral. Mas parece que pode ser provado sem análise complexa.

Artur Costa Steiner

-- 
Esta mensagem foi verificada pelo sistema de antivírus e
acredita-se estar livre de perigo.

-- 
Esta mensagem foi verificada pelo sistema de antiv�rus e
 acredita-se estar livre de perigo.



[obm-l] Geometria

2018-04-12 Por tôpico Douglas Oliveira de Lima
Caros amigos , tenho um problema bem legal e estou compartilhando. Ai vai:

Numa reta marcam-se os pontos A,B,C,D nesta ordem , e no mesmo semiplano
constroem-se os triângulos equiláteros ABP, BCQ e CDR de lados 5, 3 e x
respectivamente, sendo o angulo PQR igual a 120 graus, determine x.



Será que teria alguma construção bonita para solucionå-lo?

Abraco
Douglas Oliveira.

-- 
Esta mensagem foi verificada pelo sistema de antiv�rus e
 acredita-se estar livre de perigo.



[obm-l] Combinatória

2018-04-10 Por tôpico Douglas Oliveira de Lima
Caros amigos , retomando o raciocinio, rs, estou com um problema um tanto
interessante que nao sei como fazer:

Existe algum jeito de calcular o valor do somatório dos produtos
n(k).(101-k) onde k varia de 1 a 98 e n(k) é o número de divisores de k.


Qualquer ajuda será bem vinda.


Abraco do
Douglas Oliveira.

-- 
Esta mensagem foi verificada pelo sistema de antiv�rus e
 acredita-se estar livre de perigo.



Re: [obm-l] Geometria plana

2018-04-02 Por tôpico Douglas Oliveira de Lima
Da para fazer uma prova por absurdo.
Fica bom, suponha que a reta nao tangencia a circunferencia entao trace a
tangente e vai chegar em um absurdo.

Abraco
Douglas Oliveira.

Em seg, 2 de abr de 2018 11:14, Claudio Arconcher <arclaud...@hotmail.com>
escreveu:

> Bom dia caros colegas.
>
> Ponhamos ABCD o quadrado (o ponto A está no lado de baixo e à esquerda,
> segue-se o ponto B à direita, C e D estão no lado de acima fechando o
> circuito ABCD ).
>
> Ponhamos: AP=x e AQ=y, segue-se, QD=1-y e PB=1-x.
>
> Tracemos a circunferência de centro C e raio 1, ela tangencia AD em D e AB
> em B, agora seja M um ponto no quarto dessa circunferência interno ao
> quadrado ABCD e tracemos a tangente a ela por M, cortando AD em Q e AB em P
> ( serão, de fato os pontos esperados ), tem-se: QD=1-y = QM e PB=1 – x =
> PM, o perímetro do triângulo retângulo QAP é igual a 2. Reciprocamente se
> consideramos o triângulo AQP de perímetro 2 fixado antes o ponto M será o
> mesmo, todos esses triângulos são assim obtidos, com PQ tangente à
> circunferência em um ponto M com a propriedade descrita.
>
> Agora basta examinar as congruências dos triângulos retângulos CDQ e CMQ
> e, também, CBP e CMP, isso nos leva a concluir que o ângulo PCQ mede 45 º.
>
> Espero que o “coelhinho da Páscoa” concorde comigo.
>
> Abraço.
>
> Cláudio.
>
>
>
> *De:* owner-ob...@mat.puc-rio.br [mailto:owner-ob...@mat.puc-rio.br] *Em
> nome de *Douglas Oliveira de Lima
> *Enviada em:* domingo, 1 de abril de 2018 17:25
> *Para:* obm-l@mat.puc-rio.br
> *Assunto:* [obm-l] Geometria plana
>
>
>
> Olá amigos, pra quem gosta de geometria plana, compartilhando aqui uma
> questão do coelhinho da páscoa que achei legal.
>
>
>
> 1) Em um quadrado ABCD de lado unitário tomam-se os pontos P e Q sobre os
> lados AB e AD respectivamente, de modo que o perímetro do triângulo APQ
> seja igual a 2. Calcule a medida do ângulo  PCQ.
>
>
>
> Um abraço
>
>
>
> Douglas Oliveira.
>
>
> --
> Esta mensagem foi verificada pelo sistema de antiv�rus e
> acredita-se estar livre de perigo.
>
>
> <https://www.avast.com/sig-email?utm_medium=email_source=link_campaign=sig-email_content=emailclient>
>  Livre
> de vírus. www.avast.com
> <https://www.avast.com/sig-email?utm_medium=email_source=link_campaign=sig-email_content=emailclient>.
> <#m_3844324294932745576_DAB4FAD8-2DD7-40BB-A1B8-4E2AA1F9FDF2>
>
> --
> Esta mensagem foi verificada pelo sistema de antivírus e
> acredita-se estar livre de perigo.
>

-- 
Esta mensagem foi verificada pelo sistema de antiv�rus e
 acredita-se estar livre de perigo.



Re: [obm-l] Geometria plana

2018-04-02 Por tôpico Douglas Oliveira de Lima
Entao Claudio, eu pensei assim tb, mas a parte do reciprocamente, me deixa
incomodado, pois se o perimetro for 2 como provar que a circunferencia
tangencia em M.

Douglas Oliveira.

Em seg, 2 de abr de 2018 11:14, Claudio Arconcher <arclaud...@hotmail.com>
escreveu:

> Bom dia caros colegas.
>
> Ponhamos ABCD o quadrado (o ponto A está no lado de baixo e à esquerda,
> segue-se o ponto B à direita, C e D estão no lado de acima fechando o
> circuito ABCD ).
>
> Ponhamos: AP=x e AQ=y, segue-se, QD=1-y e PB=1-x.
>
> Tracemos a circunferência de centro C e raio 1, ela tangencia AD em D e AB
> em B, agora seja M um ponto no quarto dessa circunferência interno ao
> quadrado ABCD e tracemos a tangente a ela por M, cortando AD em Q e AB em P
> ( serão, de fato os pontos esperados ), tem-se: QD=1-y = QM e PB=1 – x =
> PM, o perímetro do triângulo retângulo QAP é igual a 2. Reciprocamente se
> consideramos o triângulo AQP de perímetro 2 fixado antes o ponto M será o
> mesmo, todos esses triângulos são assim obtidos, com PQ tangente à
> circunferência em um ponto M com a propriedade descrita.
>
> Agora basta examinar as congruências dos triângulos retângulos CDQ e CMQ
> e, também, CBP e CMP, isso nos leva a concluir que o ângulo PCQ mede 45 º.
>
> Espero que o “coelhinho da Páscoa” concorde comigo.
>
> Abraço.
>
> Cláudio.
>
>
>
> *De:* owner-ob...@mat.puc-rio.br [mailto:owner-ob...@mat.puc-rio.br] *Em
> nome de *Douglas Oliveira de Lima
> *Enviada em:* domingo, 1 de abril de 2018 17:25
> *Para:* obm-l@mat.puc-rio.br
> *Assunto:* [obm-l] Geometria plana
>
>
>
> Olá amigos, pra quem gosta de geometria plana, compartilhando aqui uma
> questão do coelhinho da páscoa que achei legal.
>
>
>
> 1) Em um quadrado ABCD de lado unitário tomam-se os pontos P e Q sobre os
> lados AB e AD respectivamente, de modo que o perímetro do triângulo APQ
> seja igual a 2. Calcule a medida do ângulo  PCQ.
>
>
>
> Um abraço
>
>
>
> Douglas Oliveira.
>
>
> --
> Esta mensagem foi verificada pelo sistema de antiv�rus e
> acredita-se estar livre de perigo.
>
>
> <https://www.avast.com/sig-email?utm_medium=email_source=link_campaign=sig-email_content=emailclient>
>  Livre
> de vírus. www.avast.com
> <https://www.avast.com/sig-email?utm_medium=email_source=link_campaign=sig-email_content=emailclient>.
> <#m_3844324294932745576_DAB4FAD8-2DD7-40BB-A1B8-4E2AA1F9FDF2>
>
> --
> Esta mensagem foi verificada pelo sistema de antivírus e
> acredita-se estar livre de perigo.
>

-- 
Esta mensagem foi verificada pelo sistema de antiv�rus e
 acredita-se estar livre de perigo.



[obm-l] Geometria plana

2018-04-01 Por tôpico Douglas Oliveira de Lima
Olá amigos, pra quem gosta de geometria plana, compartilhando aqui uma
questão do coelhinho da páscoa que achei legal.

1) Em um quadrado ABCD de lado unitário tomam-se os pontos P e Q sobre os
lados AB e AD respectivamente, de modo que o perímetro do triângulo APQ
seja igual a 2. Calcule a medida do ângulo  PCQ.

Um abraço

Douglas Oliveira.

-- 
Esta mensagem foi verificada pelo sistema de antiv�rus e
 acredita-se estar livre de perigo.



[obm-l] Functional equation(ajuda)

2018-03-31 Por tôpico Douglas Oliveira de Lima
Olá caros amigos, preciso de uma ajuda para resolver os seguintes problemas:

1) Uma função f:N*-->N* é tal que 0<=f(1)<204 e, para todo n>0, tem-se que
f(n+1)=(n/2004 +1/n)[f(n)]^2-(n^3)/2004 +1.
A quantidade de elementos da imagem de f que são números primos é:

2)Sejam u e v números reais tais que IuI<=3, IvI<=2. Determine o valor
mínimo de
f(u,v)=(u-v)^2+[((144-16u^2)^(1/2))/3 - (4-v^2)^(1/2)]^2.

Forte abraço.

Douglas Oliveira.

-- 
Esta mensagem foi verificada pelo sistema de antiv�rus e
 acredita-se estar livre de perigo.



Re: [obm-l] probleminhas de geometria

2018-03-28 Por tôpico Douglas Oliveira de Lima
Entao. acho que para qualquer circunferencia(concentrica ) sai usando
complexos, vamos ver,

O valor pedido será (w-Z1)(w-z1)+(w-Z2)(w-z2)+(w-Z3)(w-z3)=A, onde z1 é o
conjugado de Z1.

Podemos representar a circunferencia por modulo de w igual a r e o
triangulo equilatero por z^3-k^3=0 .

Assim o valor de A será 3r^2+3k^2-w(Z1+z1+Z2+z2+Z3+z3) logo A=3r^2+3k^2.

Pronto morreu.


Um abraco
 Douglas Oliveira.
Mas o valor de A será


Em 27 de mar de 2018 12:06, "Claudio Buffara" 
escreveu:

Achei estes dois bonitinhos:

1) Prove que, sendo P um ponto qualquer da circunferência inscrita a um
triângulo equilátero ABC, PA^2 + PB^2 + PC^2 é constante.
1A) Prove que isso vale para qualquer circunferência concêntrica com o
incírculo (tem uma demonstração legal para o circumcírculo usando o teorema
de Ptolomeu).


2) Um bolo tem a forma de um paralelepípedo retângulo de base quadrada e
tem cobertura no topo e nas quatro faces.
Mostre como dividir o bolo entre 7 pessoas de modo que cada um receba a
mesma quantidade de bolo e de cobertura.

Obs: a solução que envolve bater o bolo num liquidificador e dividir a
gororoba resultante em 7 partes de mesmo peso não é válida.

[]s,
Claudio.



-- 
Esta mensagem foi verificada pelo sistema de antivírus e
acredita-se estar livre de perigo.

-- 
Esta mensagem foi verificada pelo sistema de antiv�rus e
 acredita-se estar livre de perigo.



[obm-l] Re: [obm-l] Re: [obm-l] Teoria dos números

2018-03-19 Por tôpico Douglas Oliveira de Lima
E (-1,-1,2) e suas permutacoes.

Em 19 de mar de 2018 10:25, "Pedro José" <petroc...@gmail.com> escreveu:

> Bom dia!
>
> Poderia postar a solução? Não consegui achar nenhuma restrição para
> trabalhar num subconjunto  pequeno dos inteiros.
> Creio que vá ser apenas a trivial (0,0,1) e suas permutações.
>
> grato,
> PJMS
>
> Em 13 de março de 2018 20:19, Douglas Oliveira de Lima <
> profdouglaso.del...@gmail.com> escreveu:
>
>> Essa achei legal e estou postando.
>>
>> *Resolva nos inteiros a seguinte equação:  (x + y)(y + z)(z + x)/2 + (x +
>> y + z)3 = 1 – xyz* .
>>
>> Abraço do
>> Douglas Oliveira
>>
>> --
>> Esta mensagem foi verificada pelo sistema de antivírus e
>> acredita-se estar livre de perigo.
>
>
>
> --
> Esta mensagem foi verificada pelo sistema de antivírus e
> acredita-se estar livre de perigo.

-- 
Esta mensagem foi verificada pelo sistema de antiv�rus e
 acredita-se estar livre de perigo.



[obm-l] Teoria dos números

2018-03-13 Por tôpico Douglas Oliveira de Lima
Essa achei legal e estou postando.

*Resolva nos inteiros a seguinte equação:  (x + y)(y + z)(z + x)/2 + (x + y
+ z)3 = 1 – xyz* .

Abraço do
Douglas Oliveira

-- 
Esta mensagem foi verificada pelo sistema de antiv�rus e
 acredita-se estar livre de perigo.



[obm-l] Ajuda em geometria e álgebra.

2018-03-13 Por tôpico Douglas Oliveira de Lima
Olá amigos, não consigo fazer esse problema por construção, já fiz por lei
dos senos e
pelo geogebra e deu 18 graus.

Eis o problema:

6 Seja D um ponto sobre o lado BC de um triângulo ABC. Supondo que, AC=BD
e o ângulo  ADC=30 graus e ACB= 48 graus , determine  a medida do ângulo
ABC.



Qualquer ajuda será bem vinda.

Abraço do

Douglas Oliveira.

-- 
Esta mensagem foi verificada pelo sistema de antiv�rus e
 acredita-se estar livre de perigo.



[obm-l] Re: [obm-l] Re: [obm-l] Álgebra

2018-03-13 Por tôpico Douglas Oliveira de Lima
Valeu Ralph, Valeu Matheus , muito obrigado.
 Tinha mesmo pensado em algo semelhante, pensei da seguinte forma:
Quando a, b ou c são zero então a expressão dá zero, logo existe abc como
fator, daí,
a expressão remanescente de grau 2 assumiria a forma
x(a^2+b^2+c^2)+y(ab+ac+bc),
e substituindo valores acha-se x e y.

Mas de qualquer forma obrigadaço.

Forte abraço do
Douglas Oliveira.

Em 13 de março de 2018 19:16, Ralph Teixeira <ralp...@gmail.com> escreveu:

> Sim! Dá 80abc(a²+b²+c²)!
>
> ...
>
> ...
>
> Ah, você quer o JEITO... Huh... é bom, er... taquei no Scientific
> Workplace e mandei ele simplificar tudo desculpa. Talvez esteja até
> correto. :P
>
> Mas com a resposta em mãos alguém vai arrumar uma maneira bonita e
> criativa de chegar na mesmaresposta no braço, né? Né? Né?
>
> ...
>
> :D
>
> Abraços preguiçosos, Ralph.
>
> P.S.: Deve ter um jeito óbvio de ver que só os termos do tipo 3,1,1 ficam.
> Ah, sim: a expressão é ímpar em cada uma das variáveis, então todos os
> expoentes de cada variável têm que ser ímpares na resposta. Mas o polinômio
> é homogêneo, ou seja, a soma dos expoentes de cada termo é 5, então todos
> os termos são da forma a^m.b^n.c^p onde m+n+p=5 são ímpares. Acho que só
> 3+1+1 satisfaz ambas as condições? Como a expressão é invariante por
> permutação de variáveis, então só haverá um coeficiente, multiplicando os
> três monômios a^3bc, ab^3c, e abc^3, ou seja, já sei que tem que dar algo
> do tipo Kabc(a^2+b^2+c^2). Para achar K, taque a=b=c=1, e calibre K. Hm,
> acho que resolveu!
>
> 2018-03-13 18:51 GMT-03:00 Douglas Oliveira de Lima <
> profdouglaso.del...@gmail.com>:
>
>> Olá meus amigos, vocês conhecem um jeito bom de simplificar isso
>> (a+b+c)^5-(a-b+c)^5-(a+b-c)^5-(b+c-a)^5
>>
>> Abraços
>> Douglas Oliveira
>>
>> --
>> Esta mensagem foi verificada pelo sistema de antivírus e
>> acredita-se estar livre de perigo.
>
>
>
> --
> Esta mensagem foi verificada pelo sistema de antivírus e
> acredita-se estar livre de perigo.

-- 
Esta mensagem foi verificada pelo sistema de antiv�rus e
 acredita-se estar livre de perigo.



[obm-l] Álgebra

2018-03-13 Por tôpico Douglas Oliveira de Lima
Olá meus amigos, vocês conhecem um jeito bom de simplificar isso
(a+b+c)^5-(a-b+c)^5-(a+b-c)^5-(b+c-a)^5

Abraços
Douglas Oliveira

-- 
Esta mensagem foi verificada pelo sistema de antiv�rus e
 acredita-se estar livre de perigo.



[obm-l] Re: [obm-l] Problema de minimização

2018-03-11 Por tôpico Douglas Oliveira de Lima
Seja o quadrilátero ABCD cujas diagonais são AC e BD, e O o ponto de
intersecção das diagonais.
Seja também um ponto P em seu interior e as distâncias PA, PB, PC, PD,
temos por desigualdade triângular
que PA+PC>=AC e PB+PD>=BD. Claramente vemos que o ponto P coincide com o
ponto O quando a soma das diagonais
coincide com a igualdade. Desta forma o ponto procurado é o encontro das
diagonais.


Forte abraço.
Douglas Oliveira.

Em 10 de março de 2018 21:07, Claudio Buffara 
escreveu:

> Aqui vai um bonitinho que eu nunca tinha visto:
>
> Dado um quadrilátero convexo, determine o ponto cuja soma das distâncias
> aos vértices do quadrilátero é mínima.
>
> Interessante que quando a distância entre dois vértices adjacentes dados
> tende a zero (e o quadrilátero “tende” a um triângulo), o ponto de mínimo
> não parece tender ao ponto de Fermat do triângulo (exceto quando o
> triângulo tem um ângulo >= 120 graus.
>
> Abs,
> Claudio.
>
> Enviado do meu iPhone
> --
> Esta mensagem foi verificada pelo sistema de antivírus e
>  acredita-se estar livre de perigo.
>
>
> =
> Instru�ões para entrar na lista, sair da lista e usar a lista em
> http://www.mat.puc-rio.br/~obmlistas/obm-l.html
> =
>

-- 
Esta mensagem foi verificada pelo sistema de antiv�rus e
 acredita-se estar livre de perigo.



[obm-l] Re: [obm-l] Re: [obm-l] Re: [obm-l] Re: [obm-l] Triângulo quase Russo - 12º - 18º - 42º

2018-03-01 Por tôpico Douglas Oliveira de Lima
Na vdd acho que confundi esse problema com outro sinistro rs.

Ah mas ta valendo, pelo menos agora agente tem outro.

Abracos.

Em 1 de mar de 2018 11:41, "Jeferson Almir" 
escreveu:

> Opa !! Deu um valor legal. Eu tinha errado a resposta é 48º. Desculpem
>
> Em qui, 1 de mar de 2018 às 11:27, Jeferson Almir <
> jefersonram...@gmail.com> escreveu:
>
>> Eu coloquei no Geogebra e deu 48,71º. Deve ter algo errado
>>
>> Em qua, 28 de fev de 2018 às 21:46, Anderson Torres <
>> torres.anderson...@gmail.com> escreveu:
>>
>>> Em 28 de fevereiro de 2018 11:59, Claudio Buffara
>>>  escreveu:
>>> > Sugestão 1: usando régua e transferidor, desenhe uma figura tão grande
>>> e
>>> > precisa quanto puder (por exemplo, ocupando a maior parte de uma folha
>>> de
>>> > A4).
>>> > Daí, meça o ângulo EDB com o transferidor e obtenha uma conjectura.
>>> > Já será um progresso: ao invés de ter que determinar o valor do ângulo
>>> e
>>> > provar que seu raciocínio está correto, você precisará apenas provar
>>> sua
>>> > conjectura.
>>> >
>>> > Sugestão 2: Como 12 = 360/30, considere um polígono regular convexo de
>>> 30
>>> > lados inscrito numa circunferência de centro A e tal que B e C sejam
>>> > vértices (adjacentes) do polígono.
>>>
>>> Tô tentando resolver dessa forma, mas acredito ser mais promissor usar
>>> um 15-ágono em que
>>> os três vértices estão na circunferência circunscrita ao triângulo.
>>> Dessa forma é mais fácil ver certas simetrias.
>>> Por exemplo, as retas que definem os ângulos inferiores atingem
>>> meios-arcos interessantes. Daí fica mais
>>> fácil verificar algumas propriedades.
>>>
>>> > Considere os vértices P e Q do polígono tais que PAB, BAC e CAQ são
>>> ângulos
>>> > adjacentes (digamos com P, B, C, Q tomados no sentido anti-horário
>>> sobre a
>>> > circunferência), com PAB = 84 graus e CAQ = 36 graus. Prove que BQ
>>> > intersecta AC em D e CP intersecta AB em E.
>>> > Será que a reta suporte de DE intersecta a circunferência em pontos
>>> que são
>>> > vértices do polígono de 30 lados?
>>> > Aqui está outra situação em que um desenho bem feito (agora também com
>>> um
>>> > compasso) pode ajudar.
>>> > Ou então, se você usar Geogebra ou algum outro software de geometria
>>> > dinâmica...
>>> >
>>> > []s,
>>> > Claudio.
>>> >
>>> >
>>> > 2018-02-28 7:36 GMT-03:00 Jeferson Almir :
>>> >>
>>> >> Queria uma ajuda nesse problema de preferência por geometria
>>> sintética :)
>>> >>
>>> >> Seja um triângulo ABC isósceles de base BC sendo  A = 12º e os pontos
>>> E e
>>> >> D sobre AB e BC respectivamente tal que os ângulos ECB= 42º e DBC
>>> =18º.
>>> >> Calcule o ângulo EDB.
>>> >>
>>> >> --
>>> >> Esta mensagem foi verificada pelo sistema de antivírus e
>>> >> acredita-se estar livre de perigo.
>>> >
>>> >
>>> >
>>> > --
>>> > Esta mensagem foi verificada pelo sistema de antivírus e
>>> > acredita-se estar livre de perigo.
>>>
>>> --
>>> Esta mensagem foi verificada pelo sistema de antivírus e
>>>  acredita-se estar livre de perigo.
>>>
>>>
>>> 
>>> =
>>> Instru�ões para entrar na lista, sair da lista e usar a lista em
>>> http://www.mat.puc-rio.br/~obmlistas/obm-l.html
>>> 
>>> =
>>>
>>
> --
> Esta mensagem foi verificada pelo sistema de antivírus e
> acredita-se estar livre de perigo.

-- 
Esta mensagem foi verificada pelo sistema de antiv�rus e
 acredita-se estar livre de perigo.



[obm-l] Re: [obm-l] Re: [obm-l] Re: [obm-l] Re: [obm-l] Triângulo quase Russo - 12º - 18º - 42º

2018-03-01 Por tôpico Douglas Oliveira de Lima
Eis a solução, quem me apresentou esse problema pela primeira vez foi meu
professor da UERJ Paulo César em 2003 se não me engano..
E depois peguei a revista que tinha a resolução com um grande amigo que
faleceu "Gandhi" Antonio Luis dos Santos.

O link da solução é
http://www.mat.puc-rio.br/~obmlistas/obm-l.200505/msg00212.html

Já postado pelo Nicolau tem tempo.

Vale a pena ler a revista é realmente muito boa, fala a respeito de 53
triplas de inteiros que satisfazem esse triângulo.

Forte abraço do
Douglas Oliveira.

Em 1 de março de 2018 11:31, Jeferson Almir 
escreveu:

> Opa !! Deu um valor legal. Eu tinha errado a resposta é 48º. Desculpem
>
> Em qui, 1 de mar de 2018 às 11:27, Jeferson Almir <
> jefersonram...@gmail.com> escreveu:
>
>> Eu coloquei no Geogebra e deu 48,71º. Deve ter algo errado
>>
>> Em qua, 28 de fev de 2018 às 21:46, Anderson Torres <
>> torres.anderson...@gmail.com> escreveu:
>>
>>> Em 28 de fevereiro de 2018 11:59, Claudio Buffara
>>>  escreveu:
>>> > Sugestão 1: usando régua e transferidor, desenhe uma figura tão grande
>>> e
>>> > precisa quanto puder (por exemplo, ocupando a maior parte de uma folha
>>> de
>>> > A4).
>>> > Daí, meça o ângulo EDB com o transferidor e obtenha uma conjectura.
>>> > Já será um progresso: ao invés de ter que determinar o valor do ângulo
>>> e
>>> > provar que seu raciocínio está correto, você precisará apenas provar
>>> sua
>>> > conjectura.
>>> >
>>> > Sugestão 2: Como 12 = 360/30, considere um polígono regular convexo de
>>> 30
>>> > lados inscrito numa circunferência de centro A e tal que B e C sejam
>>> > vértices (adjacentes) do polígono.
>>>
>>> Tô tentando resolver dessa forma, mas acredito ser mais promissor usar
>>> um 15-ágono em que
>>> os três vértices estão na circunferência circunscrita ao triângulo.
>>> Dessa forma é mais fácil ver certas simetrias.
>>> Por exemplo, as retas que definem os ângulos inferiores atingem
>>> meios-arcos interessantes. Daí fica mais
>>> fácil verificar algumas propriedades.
>>>
>>> > Considere os vértices P e Q do polígono tais que PAB, BAC e CAQ são
>>> ângulos
>>> > adjacentes (digamos com P, B, C, Q tomados no sentido anti-horário
>>> sobre a
>>> > circunferência), com PAB = 84 graus e CAQ = 36 graus. Prove que BQ
>>> > intersecta AC em D e CP intersecta AB em E.
>>> > Será que a reta suporte de DE intersecta a circunferência em pontos
>>> que são
>>> > vértices do polígono de 30 lados?
>>> > Aqui está outra situação em que um desenho bem feito (agora também com
>>> um
>>> > compasso) pode ajudar.
>>> > Ou então, se você usar Geogebra ou algum outro software de geometria
>>> > dinâmica...
>>> >
>>> > []s,
>>> > Claudio.
>>> >
>>> >
>>> > 2018-02-28 7:36 GMT-03:00 Jeferson Almir :
>>> >>
>>> >> Queria uma ajuda nesse problema de preferência por geometria
>>> sintética :)
>>> >>
>>> >> Seja um triângulo ABC isósceles de base BC sendo  A = 12º e os pontos
>>> E e
>>> >> D sobre AB e BC respectivamente tal que os ângulos ECB= 42º e DBC
>>> =18º.
>>> >> Calcule o ângulo EDB.
>>> >>
>>> >> --
>>> >> Esta mensagem foi verificada pelo sistema de antivírus e
>>> >> acredita-se estar livre de perigo.
>>> >
>>> >
>>> >
>>> > --
>>> > Esta mensagem foi verificada pelo sistema de antivírus e
>>> > acredita-se estar livre de perigo.
>>>
>>> --
>>> Esta mensagem foi verificada pelo sistema de antivírus e
>>>  acredita-se estar livre de perigo.
>>>
>>>
>>> 
>>> =
>>> Instru�ões para entrar na lista, sair da lista e usar a lista em
>>> http://www.mat.puc-rio.br/~obmlistas/obm-l.html
>>> 
>>> =
>>>
>>
> --
> Esta mensagem foi verificada pelo sistema de antivírus e
> acredita-se estar livre de perigo.
>

-- 
Esta mensagem foi verificada pelo sistema de antiv�rus e
 acredita-se estar livre de perigo.



Re: [obm-l] Como calcular?

2018-03-01 Por tôpico Douglas Oliveira de Lima
Então, esse problema é bem interessante, se eu não me engano,
ele tem sua origem com o matemático indiano Ramanujam, em um
de seus escritos.

Mas tem uma solução legal na dissertação do meu camarada Carlos Victor, do
PROFMAT,
veja: https://sca.profmat-sbm.org.br/sca_v2/get_tcc3.php?id=27919 , é o
problema de número 49.

Valeu forte abraço do
Douglas Oliveira.

Em 1 de março de 2018 10:08, Bernardo Freitas Paulo da Costa <
bernardo...@gmail.com> escreveu:

> 2018-03-01 0:56 GMT-03:00 Gabriel Tostes :
> > Define a sequencia A_(n+1)= [ (A_n)^2 - 1 ] / n (1)
> > Então A_2= sqrt(1+2A3)=sqrt(1+2(sqrt(1+3A4))... Realimentando sempre
> (substituindo A_n=sqrt(1+ nA_n+1)
> > vemos que A2 se iguala a x se lim n->oo da raiz 2^(n-2) de An é 0.
>
> Eu não entendi esta afirmação "A2 se iguala a x se lim ... = 0".  Como
> você mostra isso?  Além do mais, a sequência de raizes podia
> (podia...) tender a infinito.  Acho que também tem que mostrar que não
> é o caso.  Enfim, o que você escreveu (pode ser que você queira dizer
> outra coisa) é que "se o limite é zero, então A2 é finito", mas o que
> você precisa (para o argumento abaixo) é "se A2 é finito, então o
> limite é zero".
>
> > Seja An=n+1 + Bn. Bn outra sequencia.
>
> Realmente, essa transformação é mágica.  Eu chutei o limite (usando um
> computador) e daí calculei os outros termos, vi An = n+1, e fui provar
> que dava.  O que eu usei foi uma sequência dupla, T(n,m) = raiz(1 +
> n*raiz(1 + (n+1)*raiz(1 +  raiz(1 + m) ... ))).  Claro que
> T(n,m+1) > T(n,m), portanto lim T(n,m) existe (ou é infinito).  E para
> provar que não é infinito eu usei que o limite deveria dar T(n,inf) =
> n+1, e provei que T(n,m) < n+1 para todo m...
>
> > Então, de 1:
> > n+2+B_(n+1)=n+2 + 2Bn + ( Bn^2 + 2Bn)/n -> B_(n+1) >= 2Bn, uma inducao
> simples traz que:
> > Bn>=2^(n-2).B2
> > Entao o limite quando n vai para o infinito da raiz 2^(n-2) de An eh
> igual a B2, ou seja, B2=0 e
> > X= A2=3
>
> --
> Bernardo Freitas Paulo da Costa
>
> --
> Esta mensagem foi verificada pelo sistema de antivírus e
>  acredita-se estar livre de perigo.
>
>
> =
> Instru�ões para entrar na lista, sair da lista e usar a lista em
> http://www.mat.puc-rio.br/~obmlistas/obm-l.html
> =
>

-- 
Esta mensagem foi verificada pelo sistema de antiv�rus e
 acredita-se estar livre de perigo.



Re: [obm-l] determine all pair of integers (x,y) such that

2018-02-24 Por tôpico Douglas Oliveira de Lima
Então... como procuramos soluções inteiras, podemos ter também soluções
negativas.

1) Vamos lá, Se x<0 então 1+2^x+2^(2x+1) é inteiro somente se x=-1  logo
1+2^x+2^(2x+1)=2, mas 2 não é quadrado perfeito.

2) Se x=0 então 1+2^x+2^(2x+1)=4 então y=2 ou y=-2.

3)Se x>0 então 2^x+2^(2x+1)=2^x(1+2^(x+1))=(y-1)(y+1), como y é ímpar e
MDC(y-1, y+1)=1, temos que y-1=k2^(x-1) ou y+1=k2^(x-1), com k>0 e ímpar.

4) Se y-1=k2^(x-1) então 1+2^(x+1)=k+(k^2)2^(x-2) logo
2^(x-2)=(k-1)/(8-k^2), desta forma
8-k^2 I k-1, então pela desigualdade I 8-k^2 I<= I k-1 I, k=3 , 2^(x-2)=-2,
no qual não há soluções.

5) Se y+1=k2^(x-1), com a mesma analogia do passo 4 teremos 2^(x-2)=4, logo
x=4 e y=23 ou y=-23

Portanto as únicas soluções serão (0,2); (0,-2); (4,23); (4,-23).



Douglas Oliveira



Em 24 de fevereiro de 2018 09:47, Luís Lopes 
escreveu:

> 1 + 2^x + 2^(2x+1) = y^2
>
>
> Sauda,c~oes,
>
>
> Recebi o problema acima de um outro grupo.
>
> Como resolver ?
>
>
> Abs,
>
> Luís
>
>
>
>
>
>
> --
> Esta mensagem foi verificada pelo sistema de antivírus e
> acredita-se estar livre de perigo.
>

-- 
Esta mensagem foi verificada pelo sistema de antiv�rus e
 acredita-se estar livre de perigo.



[obm-l] Teoria dos números :Solucões inteiras de uma equação

2017-12-15 Por tôpico Douglas Oliveira de Lima
Olá amigos , bom dia peço aos senhores uma ajuda no seguinte problema:

Dados a, b, k inteiros com k positivo e a equação x^2+axy+by^2=mt^k.

a) Determinar as condições de m para que a equação x^2+axy+by^2=mt^k tenha
soluções inteiras e encontrar as soluções quando existirem.

b) Examinar os casos k=2 e k=3.

Abraços do
DouglasOliveira.

-- 
Esta mensagem foi verificada pelo sistema de antiv�rus e
 acredita-se estar livre de perigo.



[obm-l] Probabilidade

2017-12-06 Por tôpico Douglas Oliveira de Lima
Caros amigos, preciso da ajuda dos senhores para confirmar um gabarito de
uma questão:

Eis a questão:

Num trem existem 280 animais, sendo 90 da fazenda Tampa, 110 da fazenda Boa
Vista, e 80 da fazenda Monte verde, se três dos animais fossem escolhidos
ao acaso entre os 280, qual a probabilidade de que cada um deles seja de
uma fazenda diferente?

Douglas Oliveira.

-- 
Esta mensagem foi verificada pelo sistema de antiv�rus e
 acredita-se estar livre de perigo.



Re: [obm-l] Probabilidade

2017-11-17 Por tôpico Douglas Oliveira de Lima
Valeu Pedro também achei esquisito.

Douglas Oliveira.

Em 17 de nov de 2017 16:49, "Pedro José" <petroc...@gmail.com> escreveu:

> Boa tarde!
>
> Não ficou claro o enunciado. Primeiramente cita que o lançamento é
> simultâneo, depois que Alfredo é o primeiro a jogar. tem uma vírgula
> seguida da expressão não há vencedor que não faz o menor sentido...
>
> Supondo que os lançamentos são intercalados. E que se uma pessoa atinge a
> soma 10 ganha e  o adversário não joga será:
>
> Chances favoráveis: (4,6); (6,4); (5,5) ==> 3 chances favoráveis e por
> conseguinte, 33 desfavoráveis.
>
> P10 = 1/12 e ~P10= 11/12.
>
> Para que Alfredo ganhe na segunda jogada será preciso: que Alfredo erre na
> primeira, *e* que Bernardo erre na primeira e que Alfredo acerte na
> segunda:
>
> P =(11/12)^2* (1/12)= 11^2/12^3.
>
> Supondo os lançamentos simultâneos, para o Alfredo ganhar na segunda além
> do s fatos do item anterior, ainda é necessário que o Bernardo erre a
> segunda.
>
> P* = P * 11/12 ==> P* = 11^3/12^4.
>
> Creio que seja isso. Saudações,
> PJMS
>
> Em 17 de novembro de 2017 15:03, Douglas Oliveira de Lima <
> profdouglaso.del...@gmail.com> escreveu:
>
>> Alfredo e Bernardo participam de um jogo participam de um jogo em que
>> cada um lança simultaneamente um par de dados até que um deles obtenha a
>> soma dos pontos das faces voltadas para cima igual a 10,momento em que a
>> disputa termina e o vencedor é o jogador que obteve essa soma 10,não há
>> vencedor. Se o Alfredo é o primeiro a jogar, qual é a probabilidade de que
>> ele seja o vencedor na segunda rodada (segundo lançamento de dados )
>>
>>
>>
>> --
>> Esta mensagem foi verificada pelo sistema de antivírus e
>> acredita-se estar livre de perigo.
>
>
>
> --
> Esta mensagem foi verificada pelo sistema de antivírus e
> acredita-se estar livre de perigo.

-- 
Esta mensagem foi verificada pelo sistema de antiv�rus e
 acredita-se estar livre de perigo.



[obm-l] Probabilidade

2017-11-17 Por tôpico Douglas Oliveira de Lima
Alfredo e Bernardo participam de um jogo participam de um jogo em que cada
um lança simultaneamente um par de dados até que um deles obtenha a soma
dos pontos das faces voltadas para cima igual a 10,momento em que a disputa
termina e o vencedor é o jogador que obteve essa soma 10,não há vencedor.
Se o Alfredo é o primeiro a jogar, qual é a probabilidade de que ele seja o
vencedor na segunda rodada (segundo lançamento de dados )

-- 
Esta mensagem foi verificada pelo sistema de antiv�rus e
 acredita-se estar livre de perigo.



[obm-l]

2017-10-18 Por tôpico Douglas Oliveira de Lima
Ola amigos, gostaria de uma ajuda  no seguinte problema:
Quem é maior? S=1/a+1/b+1/c ou t=a^(1/2)+b^(1/2)+c^(1/2) onde a, b e c sao
lados de um.triangulo e abc=1.

Obrigado.

-- 
Esta mensagem foi verificada pelo sistema de antiv�rus e
 acredita-se estar livre de perigo.



Re: [obm-l] soma de tan^2

2017-09-16 Por tôpico Douglas Oliveira de Lima
Eu resolvi esse problema em 2014 aqui na lista olhe
https://www.mail-archive.com/obm-l@mat.puc-rio.br/msg52281.html

Abraços.

Em 16 de set de 2017 13:23, "Carlos Gomes"  escreveu:

Olá Luis...lembro desse problema ...ele foi publicado na Mathematical
excalibur ha alguns anos https://www.math.ust.hk/excalibur/

A resposta é C(90,2)= 4005, se não me falha a memória...usa relações de
Girard num "polinômio esperto"...vou tenter ver se lembro a solução...se
lembrar ponha aqui!

Abraço, Cgomes.

Em 16 de setembro de 2017 10:48, Luís Lopes 
escreveu:

> Sauda,c~oes,
>
>
> Bom dia.
>
>
> Me mandaram a seguinte questão:
>
>
> (1) Seja S = tan²(1º) + tan²(3º) + tan²(5º) + ... + tan²(89º), calcule o
> valor de S.
>
> Como resolver ? Obrigado.
>
>
> Abs,
>
> Luís
>
>
>
> --
> Esta mensagem foi verificada pelo sistema de antivírus e
> acredita-se estar livre de perigo.
>


-- 
Esta mensagem foi verificada pelo sistema de antivírus e
acredita-se estar livre de perigo.

-- 
Esta mensagem foi verificada pelo sistema de antiv�rus e
 acredita-se estar livre de perigo.



[obm-l] Re: [obm-l] Re: [obm-l] Problema difícil.

2017-09-13 Por tôpico Douglas Oliveira de Lima
Então Bernardo, eu pensei numa parada mas não tenho certeza , pensei que os
números 997,998,999,...,1994 Não poderiam ocupar as posições de 1 a 1997,
logo pelo menos um deles ocuparia uma posição não inferior a 998, aí pensei
no 997.998=995006.

Em 12 de set de 2017 18:39, "Bernardo Freitas Paulo da Costa" <
bernardo...@gmail.com> escreveu:

> 2017-09-12 17:51 GMT-03:00 Douglas Oliveira de Lima
> <profdouglaso.del...@gmail.com>:
> > Considere a sequência de números 1,2,3,4,5,...,2017.
> > E uma certa ordenação deles a1, a2, a3, ..., a2017.
> > Agora multiplique respectivamente os números das duas sequencias
> > determinando assim uma nova sequência 1.a1, 2.a2, 3.a3, ..., 2017.a2017.
> >
> > Qual o menor valor que o maior produto da última sequência pode assumir?
>
> Esse problema não é tão difícil quanto parece.  O que você tentou fazer?
>
> Abraços,
> --
> Bernardo Freitas Paulo da Costa
>
> --
> Esta mensagem foi verificada pelo sistema de antivírus e
>  acredita-se estar livre de perigo.
>
>
> =
> Instru�ões para entrar na lista, sair da lista e usar a lista em
> http://www.mat.puc-rio.br/~obmlistas/obm-l.html
> =
>

-- 
Esta mensagem foi verificada pelo sistema de antiv�rus e
 acredita-se estar livre de perigo.



[obm-l] Problema difícil.

2017-09-12 Por tôpico Douglas Oliveira de Lima
Considere a sequência de números 1,2,3,4,5,...,2017.
E uma certa ordenação deles a1, a2, a3, ..., a2017.
Agora multiplique respectivamente os números das duas sequencias
determinando assim uma nova sequência 1.a1, 2.a2, 3.a3, ..., 2017.a2017.

Qual o menor valor que o maior produto da última sequência pode assumir?

Douglas Oliveira.

-- 
Esta mensagem foi verificada pelo sistema de antiv�rus e
 acredita-se estar livre de perigo.



Re: [obm-l] Fibonacci teoria dos numeros

2017-09-05 Por tôpico Douglas Oliveira de Lima
O problema caiu na olimpíada de matemática do Rio de Janeiro se não me
engano em 1999 ou 1998.

Em 5 de set de 2017 17:52, "Pedro José" <petroc...@gmail.com> escreveu:

> Boa tarde!
>
> O programa comera o F_28830 que é igual a zero.
> Desconsiderar o exposto anteriormente.
>
> Em 5 de setembro de 2017 16:25, Pedro José <petroc...@gmail.com> escreveu:
>
>> Boa tarde!
>>
>> Douglas,
>>
>> esse problema você viu em algum local ou foi uma conjectura sua?
>>
>> Fiz em computador a sequência de Fibonacci mod 29791, o o F_14 = 377
>> mod29791 e F_15 = 610 mod2971 e F_28844 = 377 mod29791 e F_28845 = 610
>> mod29791, o que caracteriza que haverá um padrão de repetição na geração
>> dos números de Fibonacci mod 29791. E como não tem um elemento F_i = 0 mod
>> para i <28844, não haverá mais nenhum termo F_j = 0. Pois para qualquer j
>> >= 28844 haverá um i < 28844, onde F_j = F_i <>0 mod 29791, logo 29791, não
>> tem um único múltiplo na sequência de Fibonacci e portanto, a hipótese é
>> falsa.
>>
>> Deve ter um modo mais elegante para mostrar que a proposição é fasla.
>>
>> Sds,
>> PJMS
>>
>>
>>
>>
>>
>>
>> Em 5 de setembro de 2017 10:16, Pedro José <petroc...@gmail.com>
>> escreveu:
>>
>>> Bom dia!
>>>
>>> Eu pensei que entendera, porém, os números não são sequenciais.
>>> Se nós tivermos dois números consecutivos F_j e F_j+1  congruentes
>>> módulo m, pela lei de geração da sequencia de Fibonacci teremos que F_j-1 =
>>> 0 módulo m.
>>> O enunciado deveria ter uma restrição ... existe um número de Fibonacci, não
>>> nulo, que é múltiplo n, para evitar a corrente que considera zero como
>>> o primeiro termo da sequencia, pois, aí ficaria elementar a solução.
>>> Acho que o caminho é provar que como F_1 = F_2  ==> que para algum j
>>> : F_j = F_j+1 módulo m o que leva a F_j-1= 0 módulo m.
>>> Mas por casa de pombo só não daria, por exemplo, se consideramos A_1 = 3
>>> e A_2 = 7 e Ai = A_i-1 + A_i-2, não haveria um número múltiplo de 8.
>>> A sequência mod 8 ficaria: 3, 7, 2, 1, 3, 4, 7, 3, 2, 5, 7, 4, 3, 7, 2,
>>> 1, 3, 4, 7, 3, 2, 5, 7, 4...
>>>
>>> Saudações,
>>> PJMS
>>>
>>>
>>> Em 4 de setembro de 2017 16:48, Pedro José <petroc...@gmail.com>
>>> escreveu:
>>>
>>>> Boa tarde!
>>>>
>>>> Nehab,
>>>>
>>>> não consegui entender o restante da solução, mas ele usou o sinal de
>>>> igual para congruência por comodidade de edição, e até pela lei de formação
>>>> da sequência, só o segundo e terceiro termos são iguais, quando se admite
>>>> que comece de zero, ou os dois primeiros, para a corrente que não considera
>>>> como o primeiro termo da sequencia..
>>>> Por exemplo, 13 = 23 mod 10 mas (13, 23) = 1. Portanto, não fere o
>>>> princípio de que dois números consecutivos na sequência de Fibonacci sejam
>>>> primos entre si.
>>>> Até aí captei e entendi, pelo princípio da casa de pombos. Estou
>>>> tentando entender o restante.
>>>>
>>>> Saudações,
>>>> PJMS
>>>>
>>>> Em 4 de setembro de 2017 14:53, Carlos Nehab <carlos.ne...@gmail.com>
>>>> escreveu:
>>>>
>>>>> Oi, Douglas.
>>>>>
>>>>> Acho que o mdc entre Fibbonaccis consecutivos é sempre 1...
>>>>>
>>>>> Nehab
>>>>>
>>>>>
>>>>> <https://www.avast.com/sig-email?utm_medium=email_source=link_campaign=sig-email_content=webmail>
>>>>>  Livre
>>>>> de vírus. www.avast.com
>>>>> <https://www.avast.com/sig-email?utm_medium=email_source=link_campaign=sig-email_content=webmail>.
>>>>>
>>>>> <#m_-2219119211184066607_m_-112899321461919009_m_4365848760417512581_m_408333944165922029_m_4928629599140568768_m_2006884623661450834_DAB4FAD8-2DD7-40BB-A1B8-4E2AA1F9FDF2>
>>>>>
>>>>> Em 4 de setembro de 2017 07:24, Anderson Torres <
>>>>> torres.anderson...@gmail.com> escreveu:
>>>>>
>>>>>> Em 31 de agosto de 2017 16:30, Douglas Oliveira de Lima
>>>>>> <profdouglaso.del...@gmail.com> escreveu:
>>>>>> > Olá, como posso mostrar que para algum inteiro e positivo n, existe
>>

[obm-l] Fibonacci teoria dos numeros

2017-08-31 Por tôpico Douglas Oliveira de Lima
Olá, como posso mostrar que para algum inteiro e positivo n, existe um
número de Fibonacci que é múltiplo de n?

Douglas Oliveira.

-- 
Esta mensagem foi verificada pelo sistema de antiv�rus e
 acredita-se estar livre de perigo.



[obm-l] Desigualdades

2017-08-14 Por tôpico Douglas Oliveira de Lima
Como posso prova para x,y,z positivos que
x^3+y^3+z^3+3xyz>=xy(x+y)+xz(x+z)+yz(y+z).

Douglas Oliveira .

-- 
Esta mensagem foi verificada pelo sistema de antiv�rus e
 acredita-se estar livre de perigo.



[obm-l] Teoria dos números

2017-08-14 Por tôpico Douglas Oliveira de Lima
Como posso mostrar que a sequência 1+1/2+1/3+1/4+1/5+...+1/n não é um
inteiro para n>1.

Forte abraço

Douglas Oliveira.

-- 
Esta mensagem foi verificada pelo sistema de antiv�rus e
 acredita-se estar livre de perigo.



[obm-l] Função máximo inteiro

2017-07-28 Por tôpico Douglas Oliveira de Lima
Quantas soluções positivas e inteiras possui a equação [n/10]=[n/11]+1 onde
[x] é o maior inteiro que não supera x.

Att.
Douglas Oliveira de Lima.

-- 
Esta mensagem foi verificada pelo sistema de antiv�rus e
 acredita-se estar livre de perigo.



Re: [obm-l] Geometria plana

2017-07-13 Por tôpico Douglas Oliveira de Lima
Eu fiz algo parecido , também cheguei na mesma resposta, eu cheguei na
expressão (m+n-n^2-m^2)/(m+n)(2-m-n) e tinha que maximizar isso com m e n
entre zero e um.

Obrigado.

Douglas Oliveira.

Em 12 de jul de 2017 4:10 PM, "Pedro José" <petroc...@gmail.com> escreveu:

> Boa tarde!
>
> Só faltaram as definições de a e b, a é a medida do segmento BF e b a do
> segmento CG.
>
> Desculpem-me,
> PJMS
>
> Em 12 de julho de 2017 09:08, Pedro José <petroc...@gmail.com> escreveu:
>
>> Bom dia!
>>
>> Estava indo pelo caminho errado, derivadas parciais.
>>
>> x + y = ab/(a+b) + (1-a) (1-b) / (2-(a+b)) = ((a+b) - (a^2+b^2))/ (2(a+b)
>> - (a+b)^2)
>>
>> Agora ficou fácil, basta mostrar que 2(a^2+b^2) >= (a+b)^2, o que implica
>> em x + y <= 0,5 e S(PFQG) <= 1/4
>> Mas por Cauchy-Shwarz fica clara a desigualdade e o caso particular da
>> igualdade só se dá para a = b.
>> Portanto, a=b é a condição e a área máxima é 1/4. Atentar que a e b não
>> podem assumir os valores 0 ou 1.
>>
>> Saudações,
>> PJMS
>>
>> Em 11 de julho de 2017 20:50, Pedro José <petroc...@gmail.com> escreveu:
>>
>>> Boa noite!
>>>
>>> Não consegui por completo, mas a solução é 1/4 e vale para BF=CG . BF<>0
>>> e BF <>1
>>>
>>> S(PFQG) =  S(FCD) - S(QCG) - S(PGD) ==> S(PFQG) = 1/2 - S(QCG) - S(PGD)
>>> (i)
>>>
>>> S (AGD) + S(BCG) = CG/2 +GD/2 = 1/2
>>>
>>> S(QCG) + S(PGD) + S(APD) + S(BCQ) = S (AGD) + S(BCG) = 1/2 (ii)
>>>
>>> por (i), se S(PFQG) é máximo então S(QCG) + S(PGD) é mínimo.
>>>
>>> por (ii) se S(QCG) + S(PGD) é mínimo, então S(APD) + S(BCQ) é máximo.
>>> (iii)
>>>
>>> seja x a medida da altura do triângulo BCQ, relativo ao vértice Q e y a
>>> altura do triângulo APD, relativa à P, de (iii) temos que x+ y deve ser
>>> máximo.
>>>
>>> x = ab/(a+b) e y = (1-a) (1-b) / (2-(a+b)), onde x é a medida de BF e y
>>> a medida de CG.
>>> É fácil mostrar que quando a=b ==> x+ y = 1/2 e S(PFQG) = 1/4.
>>>
>>> Difícil, pelo menos para mim, é mostrar que x + y < 0,5, quando a<>b e
>>> por conseguinte S(PFQG) < 1/4.
>>>
>>> Morri na praia.
>>>
>>> Saudações,
>>> PJMS
>>>
>>>
>>> Em 10 de julho de 2017 10:48, Douglas Oliveira de Lima <
>>> profdouglaso.del...@gmail.com> escreveu:
>>>
>>>> Sejam F e G pontos sobre AB e CD de um quadrado unitário ABCD. AG e DF
>>>> se interceptam em P,
>>>> e CF e BG se interceptam em Q. Determinar a posição dos pontos F e G
>>>> para que o quadrilátero PFQG tenha área máxima.
>>>>
>>>> Douglas Oliveira.
>>>>
>>>> --
>>>> Esta mensagem foi verificada pelo sistema de antivírus e
>>>> acredita-se estar livre de perigo.
>>>
>>>
>>>
>>
>
> --
> Esta mensagem foi verificada pelo sistema de antivírus e
> acredita-se estar livre de perigo.

-- 
Esta mensagem foi verificada pelo sistema de antiv�rus e
 acredita-se estar livre de perigo.



[obm-l] Problema de função elementar

2017-07-13 Por tôpico Douglas Oliveira de Lima
Seja F uma função crescente definida para todo número real x, 0<=x<=1, tal
que

a)  F(0)=0

b)  F(x/3)=F(x)/2

c)  F(1-x)=1-F(x)

Encontrar F(21/2017).


Douglas Oliveira

-- 
Esta mensagem foi verificada pelo sistema de antiv�rus e
 acredita-se estar livre de perigo.



[obm-l] Algebra (Polinomios)

2017-07-10 Por tôpico Douglas Oliveira de Lima
Encontrar o resto da divisão do polinomio (x^2+x+1)^40 por (x+1)^3.

Obs: Sem usar derivadas.

Douglas Oliveira.

-- 
Esta mensagem foi verificada pelo sistema de antiv�rus e
 acredita-se estar livre de perigo.



[obm-l] Álgebra (Equação funcional)

2017-07-10 Por tôpico Douglas Oliveira de Lima
Encontrar todas as funções f(x), definida nos reais, tais que

1) f(1)=1
2) f(x_1+x_2)=f(x_1)+f(x_2)
3) f(1/x)=(1/x^2).f(x), para x diferente de zero..

Douglas Oliveira

-- 
Esta mensagem foi verificada pelo sistema de antiv�rus e
 acredita-se estar livre de perigo.



Re: [obm-l] Somas iguais

2017-07-10 Por tôpico Douglas Oliveira de Lima
Queria propor um problema em cima desse, fiquei pensando que realmente é
possível de dividir em dois subgrupos,
a pergunta seria:

De quantas formas é possível dividir em dois subgrupos?

Douglas Oliveira.

Em 9 de julho de 2017 20:04, Vanderlei Nemitz 
escreveu:

> Obrigado, Pedro!
> Acho que ficou claro, sim!
>
> Em 8 de jul de 2017 3:51 PM, "Pedro Soares" 
> escreveu:
>
>> Desculpe se ficou mal escrito* heheh
>>
>>
>> 
>>  Virus-free.
>> www.avg.com
>> 
>> <#m_8448995251092151276_m_-8671497293299101645_DAB4FAD8-2DD7-40BB-A1B8-4E2AA1F9FDF2>
>>
>> 2017-07-08 15:26 GMT-03:00 Pedro Soares :
>>
>>> Para a soma de n números naturais ser par essa sequência deve possuir um
>>> número par de números impares. Logo, se está se somando de 1 a n e a soma é
>>> par  para n = 2k - 1 ou n = 2k onde k é multiplo de 2( se k for impar
>>> teremos um número impar de números impares na soma).
>>> O caso em que n=2k é trivial, pode-se pegar os extremos da soma e
>>> colocar em um subgrupo, os próximos extremos colocar no outro subgrupo e
>>> repetir essa ação k/2 vezes( lembre-se que k é multiplo de 2, então podemos
>>> fazer isso).
>>> Para n = 2k - 1 primeiro olhe para k = 2, claramente podemos separar nos
>>> subgrupos {1,2} e {3} que possuem a mesma soma.
>>> Agora suponha que vale para k = j, vamos provar que vale para k = n + 2
>>> por indução.
>>> A soma para n = 2( k + 2 ) + 1 é igual a soma para n = 2k( que vamos
>>> chamar de S(n) ) mais quatro termos consecutivos ( n+1, n+2, n+3, n+4).
>>> S(n) já sabemos dividir em subgrupos de igual soma por hipótese. Além
>>> disso, podemos alocar os termos faltantes usando a mesma estratégia usada
>>> para o caso n=2k( os termos n+1 e n+4 vão para um subgrupo e os termos n+2
>>> e n+3 vão para o outro). Logo, se vale para k = j vale k = j + 2. Como vale
>>> para k = 2 vale para todo multiplo de 2.
>>> Como já provamos para os dois casos em que separamos isso conclui nossa
>>> prova :)
>>>
>>> Desculpe se ficou mau escrito, digitei conforme fui pensando
>>>
>>>
>>> On Saturday, 8 July 2017, Vanderlei Nemitz 
>>> wrote:
>>>
 Bom dia!
 Gostaria de saber se alguém tem uma solução para esse problema:

 *Mostre que se a soma dos números de 1 até n é par, então é possível
 separar os números de 1 até n em dois subgrupos de números de igual soma.*

 Muito obrigado!

 Vanderlei

 --
 Esta mensagem foi verificada pelo sistema de antivírus e
 acredita-se estar livre de perigo.
>>>
>>>
>>
>> --
>> Esta mensagem foi verificada pelo sistema de antivírus e
>> acredita-se estar livre de perigo.
>
>
> --
> Esta mensagem foi verificada pelo sistema de antivírus e
> acredita-se estar livre de perigo.
>

-- 
Esta mensagem foi verificada pelo sistema de antiv�rus e
 acredita-se estar livre de perigo.



[obm-l] Geometria plana

2017-07-10 Por tôpico Douglas Oliveira de Lima
Sejam F e G pontos sobre AB e CD de um quadrado unitário ABCD. AG e DF se
interceptam em P,
e CF e BG se interceptam em Q. Determinar a posição dos pontos F e G para
que o quadrilátero PFQG tenha área máxima.

Douglas Oliveira.

-- 
Esta mensagem foi verificada pelo sistema de antiv�rus e
 acredita-se estar livre de perigo.



[obm-l] Geometria plana

2017-07-08 Por tôpico Douglas Oliveira de Lima
Num triângulo equilátero ABC, as cevianas BD e CE se encontram em P, se a
área do triângulo BCP é igual a área do quadrilátero ADPE ,  determine o
ângulo BPC.

Douglas Oliveira.

-- 
Esta mensagem foi verificada pelo sistema de antiv�rus e
 acredita-se estar livre de perigo.



[obm-l] Re: [obm-l] Re: [obm-l] Teoria dos números

2017-07-06 Por tôpico Douglas Oliveira de Lima
Opa , sim, é a•b•c

Em 6 de jul de 2017 11:14 PM, "Carlos Nehab" <carlos.ne...@gmail.com>
escreveu:

> Oi, Douglas,
>
> Esse "abc" é a x b x c (produto) ou o inteiro de algarismos a, b e c
> (100a+10b+c)?
>
> Abs
> Nehab
>
>
> <https://www.avast.com/sig-email?utm_medium=email_source=link_campaign=sig-email_content=webmail>
>  Livre
> de vírus. www.avast.com
> <https://www.avast.com/sig-email?utm_medium=email_source=link_campaign=sig-email_content=webmail>.
> <#m_1267597801263667645_DAB4FAD8-2DD7-40BB-A1B8-4E2AA1F9FDF2>
>
> Em 6 de julho de 2017 14:03, Douglas Oliveira de Lima <
> profdouglaso.del...@gmail.com> escreveu:
>
>> Encontrar todos os inteiros positivos a,b e c tais que a^b+b^c=abc.
>>
>> --
>> Esta mensagem foi verificada pelo sistema de antivírus e
>> acredita-se estar livre de perigo.
>
>
>
> --
> Esta mensagem foi verificada pelo sistema de antivírus e
> acredita-se estar livre de perigo.

-- 
Esta mensagem foi verificada pelo sistema de antiv�rus e
 acredita-se estar livre de perigo.



[obm-l] Teoria dos números

2017-07-06 Por tôpico Douglas Oliveira de Lima
Encontrar todos os inteiros positivos a,b e c tais que a^b+b^c=abc.

-- 
Esta mensagem foi verificada pelo sistema de antiv�rus e
 acredita-se estar livre de perigo.



Re: [obm-l] Geometria plana (Ajuda)

2017-06-28 Por tôpico Douglas Oliveira de Lima
Opa desculpe, CF é ceviana que passa por P.

Em 28 de jun de 2017 11:05 AM, "Pedro José" <petroc...@gmail.com> escreveu:

> Bom dia!
>
> O ponto F não foi definido, mas foram definidas duas medidas de ângulos
> aos quais o ponto F pertence: BCF=20 graus e FCA=40 graus.
> Não faltou definir o ponto F?
>
> Sds,
> PJMS
>
> Em 28 de junho de 2017 09:15, Douglas Oliveira de Lima <
> profdouglaso.del...@gmail.com> escreveu:
>
>> Olá meus amigos preciso de uma ajuda pra resolver a seguinte questão:
>>
>> Num triângulo ABC , tracam-se as cevianas AD e BE, que se encontram no
>> ponto P, tal que BAD= 10 graus, DAC=70 graus, BCF=20 graus e FCA=40 graus,
>> traçando a ceviana BE que passa por P e o segmento de reta que une os
>> pontos E e M, sendo M ponto médio de BC, determinar o ângulo CME.
>>
>> GRATO!!
>> Douglas Oliveira.
>>
>> --
>> Esta mensagem foi verificada pelo sistema de antivírus e
>> acredita-se estar livre de perigo.
>
>
>
> --
> Esta mensagem foi verificada pelo sistema de antivírus e
> acredita-se estar livre de perigo.

-- 
Esta mensagem foi verificada pelo sistema de antiv�rus e
 acredita-se estar livre de perigo.



[obm-l] Geometria plana (Ajuda)

2017-06-28 Por tôpico Douglas Oliveira de Lima
Olá meus amigos preciso de uma ajuda pra resolver a seguinte questão:

Num triângulo ABC , tracam-se as cevianas AD e BE, que se encontram no
ponto P, tal que BAD= 10 graus, DAC=70 graus, BCF=20 graus e FCA=40 graus,
traçando a ceviana BE que passa por P e o segmento de reta que une os
pontos E e M, sendo M ponto médio de BC, determinar o ângulo CME.

GRATO!!
Douglas Oliveira.

-- 
Esta mensagem foi verificada pelo sistema de antiv�rus e
 acredita-se estar livre de perigo.



Re: [obm-l] Radicais

2017-06-04 Por tôpico Douglas Oliveira de Lima
Opa amigo, o radical do Indiano Ramanujam, baixe um arquivo do Carlos
Victor , muito bom tem esse problema resolvido e vários outros.
Segue o link
http://cursos.ufrrj.br/posgraduacao/profmat/dissertacoes/dissertacoe/

Um abraço
Douglas Oliveira.

Em 4 de jun de 2017 3:19 PM, "Pedro Júnior" 
escreveu:

> Olá pessoal, vocês poderiam me ajudar a solucionar o problema abaixo? Já
> vi alguns bem parecidos, mas esse está me pegando...
>
> Raiz (1+2Raiz(1+3Raiz(1+4Raiz(1+...= ?
>
> Desde já agradeço
>
> --
> Esta mensagem foi verificada pelo sistema de antivírus e
> acredita-se estar livre de perigo.

-- 
Esta mensagem foi verificada pelo sistema de antiv�rus e
 acredita-se estar livre de perigo.



[obm-l] Sistema.

2017-06-04 Por tôpico Douglas Oliveira de Lima
Olá amigos, podem me dar uma ajuda no seguinte problema:
{a/b + c/d = -1, a^2 + c^2 = 1, b^2 + d^2 = 1, b^3/a + d^3/c = x},
encontrar x.

Abraços
Douglas Oliveira.

-- 
Esta mensagem foi verificada pelo sistema de antiv�rus e
 acredita-se estar livre de perigo.



Re: [obm-l] Desigualdade

2017-05-28 Por tôpico Douglas Oliveira de Lima
A_1=3

Em 28 de mai de 2017 12:44 PM, "Esdras Muniz" <esdrasmunizm...@gmail.com>
escreveu:

> Se vc colocar a1 igual a 0, 1 ou 2 vai ver queisso não é verdade. Acho que
> é verdade se |a1|>e.
>
> Em 28 de mai de 2017 11:58, "Douglas Oliveira de Lima" <
> profdouglaso.del...@gmail.com> escreveu:
>
> Então amigos, eu tive uma idéia mas não estou conseguindo concluir, vamos
> lá:
>
> Montei uma sequência e fiz a_1=3, e assim 
> [2^(1/2)].[3^(1/4)].[4^(1/8)].[5^(1/16)]...<3
> se, e somente se,
> [3^(1/4)].[4^(1/8)].[5^(1/16)]...<(3^2)/2, portanto a_2=(3^2)/2, e assim
> sucessivamente escrevi a sequência
> a_n=(a_(n-1)^2)/n, e usei um "leminha" para resolver, que é (a_n)^2>n+1, e
> assim sai fácil, só não consegui escrever
> a prova desse lema.
>
> Mas com ele sai bem facil, pois se  (a_n)^2>n+1, então (a_n)>(n+1)^(1/2),
> logo  (a_(n-1)^2)/n>(n+1)^(1/2), ou seja,
> a_(n-1)>n^(1/2)(n+1)^(1/4), ., 
> a_1>[2^(1/2)].[3^(1/4)].[4^(1/8)].[5^(1/16)]...,
> e como a_1=3, está provado.
>
> Peço a ajuda de vocÊs para provar o lema.
>
>  Lema:
> Considere a sequência a_n=(a_(n-1)^2)/n, onde a_1=3 então (a_n)^2>n+1.
>
>
>
> Douglas Oliveira
>
>
>
>
> Em 27 de maio de 2017 18:10, Esdras Muniz <esdrasmunizm...@gmail.com>
> escreveu:
>
>> Solução muito boa.
>>
>> Em 27 de mai de 2017 00:37, "Gabriel Tostes" <gtos...@icloud.com>
>> escreveu:
>>
>>> Tira ln, esse produto vai ser:
>>> Sum{n>=1} ln(n+1)/(2^n) = M
>>>
>>> Bora escrever M de outro jeito:
>>>
>>> M= ln(2) + [ln(3)-ln(2)]/2 + [ln(4)-ln(3)]/2^2 + ...
>>>
>>> M= Sum{n>=1} (ln(n+1)-ln(n))/2^(n-1)
>>>
>>> Como ln(n+1)-ln(n)=ln(1+1/n)<1/n
>>>
>>> M=2} 1/n.2^(n-1) = L + ln(2)
>>>
>>> Para achar L considere:
>>> 1/(1-x)= 1+x^2+x^3+...
>>>
>>> Integrando essa expressao temos que -(1/x).ln(1-x)= 1+x/2+x^2/3+...
>>> Substituindo x=1/2 achamos que L=2ln(2)-1
>>> E entao
>>> M< 3ln(2)-1 < ln(3)
>>>
>>>  E o produto pedido inicialmente eh menor que 3
>>>
>>>
>>>
>>>
>>>
>>>
>>>
>>>
>>> Sent from my iPad
>>> > On May 26, 2017, at 9:47 PM, Douglas Oliveira de Lima <
>>> profdouglaso.del...@gmail.com> wrote:
>>> >
>>> > Como posso fazer essa daqui:
>>> >
>>> > [2^(1/2)].[3^(1/4)].[4^(1/8)].[5^(1/16)]...<3
>>> >
>>> > Grande abraço a todos
>>> >
>>> > DouglasOliveira
>>> >
>>> > --
>>> > Esta mensagem foi verificada pelo sistema de antivírus e
>>> > acredita-se estar livre de perigo.
>>>
>>> --
>>> Esta mensagem foi verificada pelo sistema de antivírus e
>>>  acredita-se estar livre de perigo.
>>>
>>>
>>> 
>>> =
>>> Instruções para entrar na lista, sair da lista e usar a lista em
>>> http://www.mat.puc-rio.br/~obmlistas/obm-l.html
>>> 
>>> =
>>>
>>
>> --
>> Esta mensagem foi verificada pelo sistema de antivírus e
>> acredita-se estar livre de perigo.
>>
>
>
> --
> Esta mensagem foi verificada pelo sistema de antivírus e
> acredita-se estar livre de perigo.
>
>
>
> --
> Esta mensagem foi verificada pelo sistema de antivírus e
> acredita-se estar livre de perigo.

-- 
Esta mensagem foi verificada pelo sistema de antiv�rus e
 acredita-se estar livre de perigo.



Re: [obm-l] Desigualdade

2017-05-28 Por tôpico Douglas Oliveira de Lima
Então amigos, eu tive uma idéia mas não estou conseguindo concluir, vamos
lá:

Montei uma sequência e fiz a_1=3, e assim
[2^(1/2)].[3^(1/4)].[4^(1/8)].[5^(1/16)]...<3
se, e somente se,
[3^(1/4)].[4^(1/8)].[5^(1/16)]...<(3^2)/2, portanto a_2=(3^2)/2, e assim
sucessivamente escrevi a sequência
a_n=(a_(n-1)^2)/n, e usei um "leminha" para resolver, que é (a_n)^2>n+1, e
assim sai fácil, só não consegui escrever
a prova desse lema.

Mas com ele sai bem facil, pois se  (a_n)^2>n+1, então (a_n)>(n+1)^(1/2),
logo  (a_(n-1)^2)/n>(n+1)^(1/2), ou seja,
a_(n-1)>n^(1/2)(n+1)^(1/4), .,
a_1>[2^(1/2)].[3^(1/4)].[4^(1/8)].[5^(1/16)]...,
e como a_1=3, está provado.

Peço a ajuda de vocÊs para provar o lema.

 Lema:
Considere a sequência a_n=(a_(n-1)^2)/n, onde a_1=3 então (a_n)^2>n+1.



Douglas Oliveira




Em 27 de maio de 2017 18:10, Esdras Muniz <esdrasmunizm...@gmail.com>
escreveu:

> Solução muito boa.
>
> Em 27 de mai de 2017 00:37, "Gabriel Tostes" <gtos...@icloud.com>
> escreveu:
>
>> Tira ln, esse produto vai ser:
>> Sum{n>=1} ln(n+1)/(2^n) = M
>>
>> Bora escrever M de outro jeito:
>>
>> M= ln(2) + [ln(3)-ln(2)]/2 + [ln(4)-ln(3)]/2^2 + ...
>>
>> M= Sum{n>=1} (ln(n+1)-ln(n))/2^(n-1)
>>
>> Como ln(n+1)-ln(n)=ln(1+1/n)<1/n
>>
>> M=2} 1/n.2^(n-1) = L + ln(2)
>>
>> Para achar L considere:
>> 1/(1-x)= 1+x^2+x^3+...
>>
>> Integrando essa expressao temos que -(1/x).ln(1-x)= 1+x/2+x^2/3+...
>> Substituindo x=1/2 achamos que L=2ln(2)-1
>> E entao
>> M< 3ln(2)-1 < ln(3)
>>
>>  E o produto pedido inicialmente eh menor que 3
>>
>>
>>
>>
>>
>>
>>
>>
>> Sent from my iPad
>> > On May 26, 2017, at 9:47 PM, Douglas Oliveira de Lima <
>> profdouglaso.del...@gmail.com> wrote:
>> >
>> > Como posso fazer essa daqui:
>> >
>> > [2^(1/2)].[3^(1/4)].[4^(1/8)].[5^(1/16)]...<3
>> >
>> > Grande abraço a todos
>> >
>> > DouglasOliveira
>> >
>> > --
>> > Esta mensagem foi verificada pelo sistema de antivírus e
>> > acredita-se estar livre de perigo.
>>
>> --
>> Esta mensagem foi verificada pelo sistema de antivírus e
>>  acredita-se estar livre de perigo.
>>
>>
>> =
>> Instruções para entrar na lista, sair da lista e usar a lista em
>> http://www.mat.puc-rio.br/~obmlistas/obm-l.html
>> =
>>
>
> --
> Esta mensagem foi verificada pelo sistema de antivírus e
> acredita-se estar livre de perigo.
>

-- 
Esta mensagem foi verificada pelo sistema de antiv�rus e
 acredita-se estar livre de perigo.



[obm-l] Re: [obm-l] Polinômios

2017-05-27 Por tôpico Douglas Oliveira de Lima
Então:

*Podemos usar o seguinte teorema: Na divisão de um polinômio p(x) por h1(x)
o resto é r1(x); na divisão de p(x) por h2(x) o resto é r2(x); na divisão
de p(x) por h1(x).h2(x) o resto é r(x). Se r(x) é dividido por h1(x) o
resto é r1(x) e dividido por h2(x) o resto é r2(x).*

*O resto da divisão de P(x) por x4 + x2 + 1  possui de grau menor ou igual
a 3:   r(x) = ax3 + bx2 + cx + d*

*De acordo com o teorema,  ax3 + bx2 + cx + d  dividido por  x2 + x + 1
deixa resto – x + 1  e  dividido por x2 – x + 1  deixa resto 3x + 5.
Então:   i)  ax3 + bx2 + cx + d = (x2 + x + 1)(ax + e) – x + 1   =>*

*ax3 + bx2 + cx + d = ax3 + (a + e)x2 + (a + e – 1)x + e + 1*

*ii) ax3 + bx2 + cx + d = (x2 – x + 1)(ax + f) + 3x + 5   =>*

*ax3 + bx2 + cx + d = ax3 + (f – a)x2 + (a – f + 3)x + f + 5*

*\**e + 1 = f + 5   =>   e – f = 4 **\**a + e – 1 = a – f + 3   =>   e
+ f = 4   =>  e = 4   e   f = 0*

*\**d = e + 1   =>   d = 5 **\**a + e = f – a   =>   2a = – 4   =>   a
= – 2 **\**b = f – a   =>   b = 2*

*\**c = a + e – 1 = – 2 + 4 – 1   =>   c = 1**\**Ou seja:   r(x) =
– 2x3 + 2x2 + x + 5*


*Observação: O que fiz nada mais foi do que congruência aplicada a
polinômios.*


*Abraços *

*Douglas Oliveira*

Em 27 de maio de 2017 11:17, Vanderlei Nemitz 
escreveu:

> Bom dia!
>
> Alguém poderia dar uma ideia na seguinte questão? Já tentes algumas
> estratégias, mas sem êxito.
>
> Um polinômio P(x) dividido por x^2 + x + 1 dá resto -x + 1 e dividido por
> x^2 -x + 1 dá resto 3x + 5. Qual o resto da divisão de P(x) por x^4 + x^2 +
> 1?
>
> A resposta que tenho é -2x^3 + 2x^2 + x + 5.
>
> Obrigado!
>
> Vanderlei
>
> --
> Esta mensagem foi verificada pelo sistema de antivírus e
> acredita-se estar livre de perigo.

-- 
Esta mensagem foi verificada pelo sistema de antiv�rus e
 acredita-se estar livre de perigo.



[obm-l] Desigualdade

2017-05-26 Por tôpico Douglas Oliveira de Lima
Como posso fazer essa daqui:

[2^(1/2)].[3^(1/4)].[4^(1/8)].[5^(1/16)]...<3

Grande abraço a todos

DouglasOliveira

-- 
Esta mensagem foi verificada pelo sistema de antiv�rus e
 acredita-se estar livre de perigo.



Re: [obm-l] desigualdade

2017-04-30 Por tôpico Douglas Oliveira de Lima
Observe quando x=2, y=3 e z=1 a desigualdade não funciona, logo não basta
substituir x+y=a,
x+z=b e y+z=c, na verdade acho que  funciona ao "contrário" x/(x+y) + y/
(y+z) + z/(z+x) <= 2.
A não ser que seja outra questão como por exemplo:
(x+y)/z +(x+z)/y +(y+z)/x >=6 o que daria certo.

Grande abraço

Douglas Oliveira.

Em 30 de abril de 2017 10:46, marcone augusto araújo borges <
marconeborge...@hotmail.com> escreveu:

> Se x, y, z são números positivos, prove que x/(x+y) + y/ (y+z) + z/(z+x) >
> = 2
>
> --
> Esta mensagem foi verificada pelo sistema de antivírus e
> acredita-se estar livre de perigo.
>

-- 
Esta mensagem foi verificada pelo sistema de antiv�rus e
 acredita-se estar livre de perigo.



Re: [obm-l] Divisores da forma 6k + 4

2017-03-19 Por tôpico Douglas Oliveira de Lima
Então, vou tentar por um caminho aqui, qualquer coisa me corrija se faltar
algum caso:

Como 88^10=2^30.11^10, então o divisor deve asumir a forma 2^a.11^b,
portanto temos
os casos a analisar:
1) O caso e que 6q+4 é da forma 2^t, com 2<=t<=30, 6q+4=2^t, assim
3q+2=2^(t-1),
3q=2^(t-1)-2, logo 2^(t-1)=2 mod(3) ou ainda 2^t=1 mod(3), portanto t é
par, logo t=2k.
Assim de 1 a 30 temos 15 números pares.

2) O Caso e que 6q+4 é da forma 11^t, mas nsse caso não temos soluções.

3) O caso em que 6q+4=2^a.11^b, com 1<=a<=10 e 1<=b<=30, portanto
3q+2=2^(a-1).11^b,
 se a=1 então b=1 e com a>=2 teremos 3q=2^(a-1).11^b-2, então
2^(a-2).11^b=1 mod(3),
logo 2^(a+b-2)=1 mod(3), assim a+b deve ser par, o que nos dá a e b com a
mesma paridade,
e portanto temos os casos a par e b par ou a ímpar e b ímpar, ou seja
5.15+5.15=150.

Juntando os casos (1) e (2) teremos 165 divisres.

Um abraço
Douglas Oliveira.

Em 18 de março de 2017 22:16, marcone augusto araújo borges <
marconeborge...@hotmail.com> escreveu:

> Quantos divisores de 88^10 deixam resto 4 quando divididos por 6?
>
> --
> Esta mensagem foi verificada pelo sistema de antivírus e
> acredita-se estar livre de perigo.
>

-- 
Esta mensagem foi verificada pelo sistema de antiv�rus e
 acredita-se estar livre de perigo.



[obm-l] Re: [obm-l] Produto de potências(contagem)

2017-03-18 Por tôpico Douglas Oliveira de Lima
Acho que raciocínio é um pouco parecido, digamos que os expoentes dos setes
sejam a,b e c assim 7^x.7^y.7^z=7^39, logo queremos as soluções naturais dá
equação x+y+z=39 com x,y e z maiores do que ou iguais a 1 , faremos a
substituição x=a+1, y=b+1 e z=c+1 , assim a+b+c=36, portanto 38!/36!2!
=19.37=703.

Desculpe os erros , digitei do celular.
Um abraço
Douglas Oliveira.

Em 18 de mar de 2017 10:01 AM, "marcone augusto araújo borges" <
marconeborge...@hotmail.com> escreveu:

> Quantas ternas ordenadas de naturais (a,b,c) maiores que 1 são tais que
> a.b.c = 7^39?
>
> --
> Esta mensagem foi verificada pelo sistema de antivírus e
> acredita-se estar livre de perigo.
>

-- 
Esta mensagem foi verificada pelo sistema de antiv�rus e
 acredita-se estar livre de perigo.



Re: [obm-l] Um produto de 3 naturais(contagem)

2017-03-18 Por tôpico Douglas Oliveira de Lima
Então, vamos lá, eu tentei dá seguinte forma:

Fatorando o número teremos 2310=2.3.5.7.11
Logo cada número possui três possibilidades para ser "encaixado"( em a, b
ou c), desta forma teriamos 3^5 porém contamos também com números dá forma
(1,1,2310), (1,2310,1), e (2310,1,1) logo teremos 243-3=240 que dividido
por 3! Para retirar as permutas nos dá como resposta 40.

Abraços
Douglas Oliveira.

Em 18 de mar de 2017 9:58 AM, "marcone augusto araújo borges" <
marconeborge...@hotmail.com> escreveu:

> Para quantos conjuntos de inteiros positivos {a,b,c} é verdade que a.b.c =
> 2310?
>
> Alguém resolveria?Agradeço.
>
> --
> Esta mensagem foi verificada pelo sistema de antivírus e
> acredita-se estar livre de perigo.
>

-- 
Esta mensagem foi verificada pelo sistema de antiv�rus e
 acredita-se estar livre de perigo.



[obm-l] Re: [obm-l] Re: [obm-l] Re: [obm-l] Re: [obm-l] Estou tentando e não sai

2017-03-10 Por tôpico Douglas Oliveira de Lima
Olá , amigos , já tinha feito esse problema e cai na mesma duvida, se o
3,4,5 é único.
Caiu uma questão parecida no nível 2 terceira fase dá OBM que pede para
encontrar o triângulo de área mínima que possui lados inteiros e área
inteira.

Bom em relação a este problema temos como resolve-lo pelas soluções
parametricas de Brahmagupta.

Onde a=n(m^2+k^2), b=m(n^2+k^2) ,
c=(m+n)(mn-k^2) , p=mn(m+n) e
 S=mnk(m+n)(mn-k^2) onde (m,n,k)=1, mn>k^2>(m^2)n/2m+n e m>=n>=1.

Assim o que fiz foi até fácil, como o raio e igual a 1 então , S=p , assim
1=k(mn-k^2), logo só teremos k=1 e mn=2. Assim o único triângulo será o
3,4,5.

Abraços
Douglas Oliveira.


Em 8 de mar de 2017 8:22 PM, "Pedro José"  escreveu:

> Boa tarde!
>
> Novamente, sem perda de generalidade, pois ao final haverá as permutações,
> a>b>c
>
> 4(a+b+c) = a^2b+a^2c+ab^2+ac^2+b^2c+bc^2-2abc-a^3-b^3-c^3
>
> f(a,b,c) = 4(a+b+c) e g(a,b,c) = a^2b+a^2c+ab^2+ac^2+b^2c+bc^2-
> 2abc-a^3-b^3-c^3
>
> a derivada parcial de g em relação a "a" é: -3a^2 + 2(c + b)a + (c -b)^2
>
> que tem duas raízes distintas de sinais contrários, já que c>b  e a é
> positivo.
>
> então o comportamento da função é:
>
> decrescente para a  crescente para  r1 < a < r2 >0
> decrescente para a > r1.
>
> mas como a > 0  para um intervalo [ao,a1], temos as seguintes
> possibilidades:
>
> (i) monótona crescente: a1 < r2
> (ii) mónotona decrescente ao > r2
> (ii) crescente de ao a r2 e decrescente de r2 a a1, ao  r2.
>
> Como f(a,b,c) é monótona crescente em relação a "a" para todo a, e uma vez
> que g(a,b,c) cresce mais rápido que f(a,b,c) com relação a "a", se fixarmos
> b e c,
> só precisamos nos preocupar com os extremos, se f(ao,b,c) < g(ao,b,c) e
> f(a1,b,c) < g(a1,b,c) não existe solução para a pertencente a [ao,a1] e b,c
> fixos.
>
> Como o triângulo é escaleno e assumimos a>b>c,
> temos que para c e b fixos, a pode variar de b+1 a c + b - 1, portanto ao
> = b+1 e a1 = c + b - 1.
>
> Para ao temos: f(ao,b,c) = 8b + 4c + 4 e g(ao,b,c) = 2(c^2-1)b +c^2 + c -
> c^3.
>
> Como b>c temos que: g(ao,b,c) > c^3 + c^2b - 2b +c^2 + c -c^3 = c^2b -
> 2b +c^2 + c
>
> Para c>= 4 temos que g (ao,b,c) > 14 b +20 >  f(ao,b,c)
>
> f(a1,b,c) = 8b + 8c + 4
>
> g(a1,b,c) = (4c+4)b -2b +1
>
> c>= 4 ==> g(a1,b,c) > 18b +1 > 10b + 8c +1 > 8b + 8c +4.
>
> Logo c<=3.
>
> c=1, não tem solução. não há triângulos escalenos com lados inteiros sendo
> um deles igual a 1. fere a desigualdade de existência  do triângulo.
>
> c=2, só há as seguintes opções b e a = b+1.
>
> temos 6b -3 = 8b + 12 ==> não há solução para c=2.
>
> para c= 3 temos duas possibilidades c= 3 e b e a= b+1 ou c =2, b e a = b+2
>
> a= b+1, b e c = 3 ==> 8b =32 ==> a=5, b=4 e c=3.
>
> a = b+ 2, b e c = 3 ==> 2b = 25, não há solução inteira.
>
> A solução é qualquer triângulo congruente a um triângulo de lados 3, 4 e
> 5. (o triângulo pitagórico de razão da PA =1).
>
> Tentei aplicar Tartaglia e garantir que a raiz fosse inteira, mas começou
> a complicar... Tive de apelar para a derivada direcional.
> Se encontrarem uma desigualdade para garantir a solução, favor postar.
>
> Saudações,
> PJMS
>
>
>
>
> Em 8 de março de 2017 07:59, Anderson Torres  > escreveu:
>
>> Eu consegui algo que pode ajudar.
>>
>> [p(p-a)(p-b)(p-c)]^1/2 = p.r = p
>>
>> p^2 = p(p-a)(p-b)(p-c)
>>
>> p = (p-a)(p-b)(p-c)
>>
>> 8p = 2(p-a) * 2(p-b) * 2(p-c)
>>
>> 4(a+b+c) = (-a+b+c) * (a-b+c) * (a+b-c)
>>
>> Escreve A = (-a+b+c), B = (a-b+c), C = (a+b-c), assim A+B+C=a+b+c, e
>>
>> ABC = 4 (A+B+C)
>>
>> Isso dá para ir limitando com desigualdades e recorrer a tentativa e erro.
>>
>> 1/4 =  1/(AB) + 1/(AC) + 1/(BC)
>>
>> Em 6 de março de 2017 20:08, Pedro José  escreveu:
>> > permutações e não combinações.
>> >
>> > Em 6 de março de 2017 20:06, Pedro José  escreveu:
>> >>
>> >>
>> >> Boa noite!
>> >>
>> >> Fui por aí e achei:
>> >>
>> >> 4(a+b+c) = a^2b+a^2c+ab^2+ac^2+b^2c+bc^2-2abc-a^3-b^3-c^3
>> >>
>> >> Se for triângulo equilátero.
>> >>
>> >> a=b=c ==> 12a = a^3 ==> a=b=c=raiz(12), que não é inteiro.
>> >>
>> >> Se for isósceles com a<>b=c, sem perda de generalidade, pois a equação
>> é
>> >> simétrica em a,b,c.
>> >>
>> >> a^3 -2ba^2+4a + 8b =0 se a é inteiro 8b = ka , com k inteiro positivo
>> pois
>> >> a,b>0
>> >>
>> >> a^3 -(k/4)a^3 + 4a + ka = 0 ==> a^3 (1-k/4) = -a (k+4) ==> a^2 =
>> >> (k+4)/(k/4-1)
>> >>
>> >> k/4 -1 | k+4 e k/4 -1 | k-4 ==> k/4 - 1 | 8 ==> k= 36, k= 20, k = 12,
>> k= 8
>> >> e para nenhum valor de k o quociente dá um quadrado perfeito.
>> >>
>> >> Portanto o triângulo é escaleno, supondo a > b > c, sem perda de
>> >> generalidade.
>> >>
>> >> Achei a solução (5,4,3) e portanto suas combinações, só que não
>> consegui
>> >> uma restrição que mostrasse ser única.
>> >>
>> >> Vou tentara outra hora.
>> >>
>> >>
>> >>
>> >> Em 6 de março de 2017 09:08, marcone augusto araújo borges
>> >>  escreveu:
>> >>>
>> >>> Quais são os 

[obm-l] Re: [obm-l] Cálculo de determinante

2017-03-01 Por tôpico Douglas Oliveira de Lima
Muito obrigado Luís, de verdade.
Analisarei os passos, inicialmente encontrei esse determinante num livro "
Excursions in calculus" do Robert M.Young e a referência dele me levou a
procurar num livro de programação " the art of computer programming" volume
2 [263] 316.

Grande abraço
Douglas Oliveira.

Em 1 de mar de 2017 9:14 AM, "Luís Lopes"  escreveu:

> Já mandei 2 ou 3 vezes esta mensagem para a lista.
> Não sei por que ela não aparece. Tento novamente.
>
> ===
> Oi, oi Douglas,
>
> Sauda,c~oes,
>
> Achei este problema legal e fiz uma busca por
> "determinant of gcd matrix" no google.
>
> Escolhi o link
>
> http://math.stackexchange.com/questions/126/determinant-
> value-of-a-square-matrix-whose-each-entry-is-the-g-c-d-of-row-and-c
>
> que me levou a
>
> http://waset.org/publications/9996770/two-different-
> computing-methods-of-the-smith-arithmetic-determinant
>
>
> < Obs: O resultado é MT bonito, uma potência de 2.
> Verdade para n=1,2,….6. Fura para n=7.
>
> Abs,
> Luís
>
> --
> Esta mensagem foi verificada pelo sistema de antivírus e
> acredita-se estar livre de perigo.
>

-- 
Esta mensagem foi verificada pelo sistema de antiv�rus e
 acredita-se estar livre de perigo.



[obm-l] Re: [obm-l] Re: [obm-l] Cálculo de determinante.

2017-02-28 Por tôpico Douglas Oliveira de Lima
Realmente não da uma potência de 2, mas o que dá? Qual Eureka eu encontro?

Abraço do Douglas

Em 27 de fev de 2017 8:10 PM, "Anderson Torres" <
torres.anderson...@gmail.com> escreveu:

> Isso já foi respondido em uma Eureka!
> E do que me lembre, não era uma potência de dois não.
>
> Em 22 de fevereiro de 2017 23:34, Douglas Oliveira de Lima
> <profdouglaso.del...@gmail.com> escreveu:
> > Olá caros amigos não consegui pensar no seguinte problema:
> >
> > 1) Calcular o determinante de uma matriz quadrada de ordem n , onde cada
> > elemento é o MDC entre i e j.
> >
> > Obs: O resultado é MT bonito, uma potência de 2.
> >
> > Agradeço a ajuda.
> >
> > Douglas Oliveira.
> >
> >
> > --
> > Esta mensagem foi verificada pelo sistema de antivírus e
> > acredita-se estar livre de perigo.
>
> --
> Esta mensagem foi verificada pelo sistema de antivírus e
>  acredita-se estar livre de perigo.
>
>
> =
> Instru�ões para entrar na lista, sair da lista e usar a lista em
> http://www.mat.puc-rio.br/~obmlistas/obm-l.html
> =
>

-- 
Esta mensagem foi verificada pelo sistema de antiv�rus e
 acredita-se estar livre de perigo.



[obm-l] Cálculo de determinante.

2017-02-22 Por tôpico Douglas Oliveira de Lima
Olá caros amigos não consegui pensar no seguinte problema:

1) Calcular o determinante de uma matriz quadrada de ordem n , onde cada
elemento é o MDC entre i e j.

Obs: O resultado é MT bonito, uma potência de 2.

Agradeço a ajuda.

Douglas Oliveira.

-- 
Esta mensagem foi verificada pelo sistema de antiv�rus e
 acredita-se estar livre de perigo.



[obm-l] Re: [obm-l] Re: [obm-l] Intervalos de crescimento de uma função.

2017-02-22 Por tôpico Douglas Oliveira de Lima
A dúvida é essa mesmo Ralph , questão de vestibular, segunda fase dá UFF.

Em 22 de fev de 2017 7:51 PM, "Ralph Teixeira" <ralp...@gmail.com> escreveu:

> Bom, nao tem dados na sua pergunta, mas concordo que muita gente
> confunde as frases:
>
> 1. "A funcao eh crescente em [-4,-3] e em [2,3]."
> e
> 2. "A funcao eh crescente em [-4,-3]U[2,3]."
>
> que, na minha modesta opiniao, NAO significam a mesma coisa... Pode
> ser que a primeira seja verdadeira mas a segunda seja falsa. Eh isso
> que voce estah dizendo, neh?
>
> Eh uma prova de disciplina, de vestibular ou...?
>
> (Serah que fui eu que escrevi o gabarito? :O )
>
> Abraco, Ralph.
>
> 2017-02-22 7:14 GMT-03:00 Douglas Oliveira de Lima
> <profdouglaso.del...@gmail.com>:
> > Olá caros amigos, tenho uma dúvida com relaçao ao intervalo de
> crescimento
> > de uma função.
> > Peguei uma questão da prova da UFF RJ acho que de 2008 que afirma que
> > conjunto onde a função f é crescente é C=[-4,-3]U[2,3]. Eu vejo como item
> > errado pois pela definição de função 2>-3 e
> >  f(2)<f(-3) o que contraria a definição de função crescente.
> > Gostaria de uma breve opinião a respeito.
> >
> > Agradeço a ajuda.
> > Douglas Oliveira.
> >
> > --
> > Esta mensagem foi verificada pelo sistema de antivírus e
> > acredita-se estar livre de perigo.
>
> --
> Esta mensagem foi verificada pelo sistema de antivírus e
>  acredita-se estar livre de perigo.
>
>
> =
> Instru�ões para entrar na lista, sair da lista e usar a lista em
> http://www.mat.puc-rio.br/~obmlistas/obm-l.html
> =
>

-- 
Esta mensagem foi verificada pelo sistema de antiv�rus e
 acredita-se estar livre de perigo.



[obm-l] Intervalos de crescimento de uma função.

2017-02-22 Por tôpico Douglas Oliveira de Lima
Olá caros amigos, tenho uma dúvida com relaçao ao intervalo de crescimento
de uma função.
Peguei uma questão da prova da UFF RJ acho que de 2008 que afirma que
 conjunto onde a função f é crescente é C=[-4,-3]U[2,3]. Eu vejo como item
errado pois pela definição de função 2>-3 e
 f(2)

[obm-l] Re: [obm-l] Re: [obm-l] Re: Sistema de equações

2017-02-04 Por tôpico Douglas Oliveira de Lima
Tem essa daqui similar, If  x3−3x2+5x−17=0 and y3−3y2+5y+11=0, What is x +
y, if x and y are the real roots of the equations?

Em 4 de fevereiro de 2017 07:12, Carlos Gomes 
escreveu:

> Pera, então a segunda equação é  y^2 - 3y^2 + 5y = 5 ==> -2y^2+5y-5=0?
> Nesse caso essa equação não possui raízes reais. Tá estranho Marcone.
> Suspeito que há algo digitado errado! Confere aí...mesmo que esteja
> digitado asim não significa que necessariamente esteja certo!
>
> Cgomes.
>
> Em 4 de fevereiro de 2017 01:32, marcone augusto araújo borges <
> marconeborge...@hotmail.com> escreveu:
>
>> não é um sistema, mas como resolver?
>>
>> --
>> *De:* owner-ob...@mat.puc-rio.br  em nome de
>> marcone augusto araújo borges 
>> *Enviado:* sexta-feira, 3 de fevereiro de 2017 19:47
>> *Para:* obm-l@mat.puc-rio.br
>> *Assunto:* [obm-l] Sistema de equações
>>
>>
>> Como nada foi afirmado, x e y devem ser números reais
>>
>>
>> Se x^3 - 3x^2 + 5x = 1 e y^2 - 3y^2 + 5y = 5, calcule x+y
>>
>> --
>> Esta mensagem foi verificada pelo sistema de antivírus e
>> acredita-se estar livre de perigo.
>>
>> --
>> Esta mensagem foi verificada pelo sistema de antivírus e
>> acredita-se estar livre de perigo.
>>
>
>
> --
> Esta mensagem foi verificada pelo sistema de antivírus e
> acredita-se estar livre de perigo.
>

-- 
Esta mensagem foi verificada pelo sistema de antiv�rus e
 acredita-se estar livre de perigo.



[obm-l] Re: Divisibilidade.

2017-01-26 Por tôpico Douglas Oliveira de Lima
Já entendi RS, obrigado pessoal. Era bobo.

Em 26 de jan de 2017 12:34 PM, "Douglas Oliveira de Lima" <
profdouglaso.del...@gmail.com> escreveu:

> Ola amigos, preciso de uma ajuda aqui, eu vi um teorema ja faz tempo(
> alguns anos), gostaria de uma ajuda para prova-lo.
>
> Seja N o número dado e verificar se N é divisível por um número primo .
>
>
> Passo 1. Se p terminar em 3, 7 ou 9, multiplique p, respectivamente, por
> 7, 3 e 9, subtraia de 1 e divida a diferença por 10. Se p terminar em 1,
> subtraia p de 1 e divida a diferença por 10. Ambos os quocientes vamos
> designar por y.
>
>
> Passo 2. Multiplique y pelo último algarismo de N e subtraia de N sem o
> último algari smo. Se a diferença for grande, de tal maneira que não seja
> possível reconhecer facilmente se é divisível por  p, repete-se o
> processo até que seja possível reconhecer facilmente a divisão por p.
>
> Observação: Se o último algarismo da diferença vezes y for maior que a
> diferença, encerra-se o processo, e verifica se a diferença é divisível por
> p.
>
>
> Douglas Oliveira
>

-- 
Esta mensagem foi verificada pelo sistema de antiv�rus e
 acredita-se estar livre de perigo.



[obm-l] Divisibilidade.

2017-01-26 Por tôpico Douglas Oliveira de Lima
Ola amigos, preciso de uma ajuda aqui, eu vi um teorema ja faz tempo(
alguns anos), gostaria de uma ajuda para prova-lo.

Seja N o número dado e verificar se N é divisível por um número primo .


Passo 1. Se p terminar em 3, 7 ou 9, multiplique p, respectivamente, por 7,
3 e 9, subtraia de 1 e divida a diferença por 10. Se p terminar em 1,
subtraia p de 1 e divida a diferença por 10. Ambos os quocientes vamos
designar por y.


Passo 2. Multiplique y pelo último algarismo de N e subtraia de N sem o
último algari smo. Se a diferença for grande, de tal maneira que não seja
possível reconhecer facilmente se é divisível por  p, repete-se o processo
até que seja possível reconhecer facilmente a divisão por p.

Observação: Se o último algarismo da diferença vezes y for maior que a
diferença, encerra-se o processo, e verifica se a diferença é divisível por
p.


Douglas Oliveira

-- 
Esta mensagem foi verificada pelo sistema de antiv�rus e
 acredita-se estar livre de perigo.



[obm-l] Qual a maior potência?

2017-01-16 Por tôpico Douglas Oliveira de Lima
Olá amigos , gostaria de uma ajuda pra um raciocínio diferente, por log eu
já fiz.

Qual a maior potência? 4^53 ou 5^44.

-- 
Esta mensagem foi verificada pelo sistema de antiv�rus e
 acredita-se estar livre de perigo.



[obm-l] Re: [obm-l] Re: [obm-l] material interessante para o ensino de geometria analítica (vetorial)

2016-11-21 Por tôpico Douglas Oliveira de Lima
Kletenik

Em 21 de nov de 2016 3:28 PM, "Adilson Francisco da Silva" <
adilson...@gmail.com> escreveu:

> Há também um curso de geometria analítica e vetors disponibilizado pelo
> site da univesptv, segue link no YouTube
> Geometria Analítica e Vetores:
>
> http://www.youtube.com/playlist?list=PLxI8Can9yAHdmzItRKhWYl_ZsDe44PUrp
>
> Em 21 de nov de 2016 2:58 PM, "Lucas Kaue Ramos de Lima" <
> kaue_lu...@hotmail.com> escreveu:
>
>> Boa tarde,
>>
>>
>> O site do Prof Reginaldo é uma ótima referência. Segue abaixo.
>>
>>
>> http://www.mat.ufmg.br/~regi/ no lado esquerdo da tela tem a opção
>> livros. Lá você encontrará um excelente conteúdo de geometria analítica e
>> álgebra vetorial.
>>
>>
>> Abs.
>> --
>> *De:* owner-ob...@mat.puc-rio.br  em nome de
>> ilhadepaqu...@bol.com.br 
>> *Enviado:* segunda-feira, 21 de novembro de 2016 13:36:00
>> *Para:* obm-l@mat.puc-rio.br
>> *Assunto:* [obm-l] material interessante para o ensino de geometria
>> analítica (vetorial)
>>
>> Bom dia, alguém conhece algum site que contém material didático,
>> interessante, para o ensino de geom analit vetorial?
>>
>> Abraços
>> Hermann
>>
>> --
>> Esta mensagem foi verificada pelo sistema de antivírus e
>> acredita-se estar livre de perigo.
>>
>> --
>> Esta mensagem foi verificada pelo sistema de antivírus e
>> acredita-se estar livre de perigo.
>>
>
> --
> Esta mensagem foi verificada pelo sistema de antivírus e
> acredita-se estar livre de perigo.

-- 
Esta mensagem foi verificada pelo sistema de antiv�rus e
 acredita-se estar livre de perigo.



Re: [obm-l] Re: Problema de geometria.

2016-11-04 Por tôpico Douglas Oliveira de Lima
Na problema que descrevi vou escrever o que fiz.
Estou sem o acento circunflexo.

1) I e o incentro de ABC

2) BF=FI (prove isso)

3) 4.(DP)(EN)=(AI).(AI) (prove isso)

4) Usando Carnot teremos R-1+R-2+R-3=R+r, 2R=r+6

5) (c\2)(c\2)=(2R-1) Facil ver.

6) Sen(ACB\2)=r\CI=raiz(3)\IF, observe o triangulo BFI.

7) (AI)(IF)=2Rr (Prove), mas AI=2raiz(2), logo IF=Rr\raiz(2)

8)Usando 3, 6 e 7 teremos Rrr=12

9) Usando 4 e 8 chegaremos a rrr+6rr=24 que resolvendo acharemos r=4cos20
-2.

10) E cheguei a seguinte area S.S=(4cos20 -2)(4cos20)(4cos20 +2)(4cos20 +4)


Bom se nao errei nada acredito ser essa a resposta, so meu teclado que nao
esta ajudando,
mas escrevi de forma a compreender.
Forte abraco
Douglas Oliveira

Em 3 de novembro de 2016 17:55, Pedro José <petroc...@gmail.com> escreveu:

> Boa tarde!
>
> Favor postar a solução.
> Até agora, só rodando em círculos.
>
> Em 3 de novembro de 2016 14:53, Douglas Oliveira de Lima <
> profdouglaso.del...@gmail.com> escreveu:
>
>> Opa Carlos , ainda pensei em te ligar rsrsrs, mas eu achei essa raiz ai
>> sim, na equação do terceiro grau,
>> fiquei com preguiça de terminar, acho que achei o raio igual a 2co20 -2
>>  algo assim nao lembro agora,
>> é porque as respostas estão tão bonitinhas que fiquei com preguiça no
>> cosseno de 20.
>> Mas vou tentar novamente já que é isso.
>>
>> Valeu demais.
>>
>> Douglas Oliveira.
>>
>> Em 2 de novembro de 2016 23:01, victorcarlos <victorcar...@globo.com>
>> escreveu:
>>
>>> Oi Douglas,
>>> Já tinha feito está questão algum tempo atrás.
>>> A idéia é vc encontrar uma equação do terceiro grau em R. Após uma
>>> transformação,  encontra- se uma equação do terceiro grau em que o cos(20
>>> graus) é raiz. A partir daí a área fica determinada.
>>> Vou tentar reescrever e te envio.
>>> Abraços
>>> Carlos Victor.
>>>
>>>
>>> Enviado por Samsung Mobile
>>>
>>>
>>>  Mensagem original 
>>> De : Douglas Oliveira de Lima
>>> Data:02/11/2016 20:22 (GMT-03:00)
>>> Para: obm-l@mat.puc-rio.br
>>> Assunto: [obm-l] Re: Problema de geometria.
>>>
>>> Me desculpe pela ignorância, deixa eu detalhar para não gerar filosofias
>>> vãs.
>>> Dado um triângulo ABC inscrito em uma  circunferência, e os pontos
>>> médios de seus lados, M ponto médio do lado BC,
>>> N ponto médio do lado AC e P ponto médio do lado AB, considere agora os
>>> pontos médios D, E, F dos menores arcos (AB), (AC), (BC) respectivamente,
>>> se os segmentos DP=1 cm, EN=2 cm e MF=3 cm, calcule a área do triângulo.
>>>
>>>
>>> Em 2 de novembro de 2016 18:57, Douglas Oliveira de Lima <
>>> profdouglaso.del...@gmail.com> escreveu:
>>>
>>>> Olá amigos , preciso de uma ajuda na seguinte questão, na verdade a
>>>> resolução porque já tentei muita coisa, já aprendi muita coisa com ela, mas
>>>> mesmo assim não a resolvi.
>>>>
>>>> As três flechas dos três lados (cordas) de um triângulo ABC inscrito em
>>>> uma circunferência de raio R
>>>> valem 1, 2 e 3 calcular a área do triângulo.
>>>>
>>>>
>>>>
>>>> Qualquer ajuda será bem vinda. Obrigado.
>>>>
>>>> Att . Douglas Oliveira
>>>>
>>>
>>>
>>> --
>>> Esta mensagem foi verificada pelo sistema de antiv�rus e
>>> acredita-se estar livre de perigo.
>>> --
>>> Esta mensagem foi verificada pelo sistema de antivírus e
>>> acredita-se estar livre de perigo.
>>>
>>
>>
>> --
>> Esta mensagem foi verificada pelo sistema de antivírus e
>> acredita-se estar livre de perigo.
>>
>
>
> --
> Esta mensagem foi verificada pelo sistema de antivírus e
> acredita-se estar livre de perigo.
>

-- 
Esta mensagem foi verificada pelo sistema de antiv�rus e
 acredita-se estar livre de perigo.



Re: [obm-l] Re: Problema de geometria.

2016-11-03 Por tôpico Douglas Oliveira de Lima
Opa Carlos , ainda pensei em te ligar rsrsrs, mas eu achei essa raiz ai
sim, na equação do terceiro grau,
fiquei com preguiça de terminar, acho que achei o raio igual a 2co20 -2
 algo assim nao lembro agora,
é porque as respostas estão tão bonitinhas que fiquei com preguiça no
cosseno de 20.
Mas vou tentar novamente já que é isso.

Valeu demais.

Douglas Oliveira.

Em 2 de novembro de 2016 23:01, victorcarlos <victorcar...@globo.com>
escreveu:

> Oi Douglas,
> Já tinha feito está questão algum tempo atrás.
> A idéia é vc encontrar uma equação do terceiro grau em R. Após uma
> transformação,  encontra- se uma equação do terceiro grau em que o cos(20
> graus) é raiz. A partir daí a área fica determinada.
> Vou tentar reescrever e te envio.
> Abraços
> Carlos Victor.
>
>
> Enviado por Samsung Mobile
>
>
> ---- Mensagem original 
> De : Douglas Oliveira de Lima
> Data:02/11/2016 20:22 (GMT-03:00)
> Para: obm-l@mat.puc-rio.br
> Assunto: [obm-l] Re: Problema de geometria.
>
> Me desculpe pela ignorância, deixa eu detalhar para não gerar filosofias
> vãs.
> Dado um triângulo ABC inscrito em uma  circunferência, e os pontos médios
> de seus lados, M ponto médio do lado BC,
> N ponto médio do lado AC e P ponto médio do lado AB, considere agora os
> pontos médios D, E, F dos menores arcos (AB), (AC), (BC) respectivamente,
> se os segmentos DP=1 cm, EN=2 cm e MF=3 cm, calcule a área do triângulo.
>
>
> Em 2 de novembro de 2016 18:57, Douglas Oliveira de Lima <
> profdouglaso.del...@gmail.com> escreveu:
>
>> Olá amigos , preciso de uma ajuda na seguinte questão, na verdade a
>> resolução porque já tentei muita coisa, já aprendi muita coisa com ela, mas
>> mesmo assim não a resolvi.
>>
>> As três flechas dos três lados (cordas) de um triângulo ABC inscrito em
>> uma circunferência de raio R
>> valem 1, 2 e 3 calcular a área do triângulo.
>>
>>
>>
>> Qualquer ajuda será bem vinda. Obrigado.
>>
>> Att . Douglas Oliveira
>>
>
>
> --
> Esta mensagem foi verificada pelo sistema de antiv�rus e
> acredita-se estar livre de perigo.
> --
> Esta mensagem foi verificada pelo sistema de antivírus e
> acredita-se estar livre de perigo.
>

-- 
Esta mensagem foi verificada pelo sistema de antiv�rus e
 acredita-se estar livre de perigo.



[obm-l] Re: Problema de geometria.

2016-11-02 Por tôpico Douglas Oliveira de Lima
Me desculpe pela ignorância, deixa eu detalhar para não gerar filosofias
vãs.
Dado um triângulo ABC inscrito em uma  circunferência, e os pontos médios
de seus lados, M ponto médio do lado BC,
N ponto médio do lado AC e P ponto médio do lado AB, considere agora os
pontos médios D, E, F dos menores arcos (AB), (AC), (BC) respectivamente,
se os segmentos DP=1 cm, EN=2 cm e MF=3 cm, calcule a área do triângulo.


Em 2 de novembro de 2016 18:57, Douglas Oliveira de Lima <
profdouglaso.del...@gmail.com> escreveu:

> Olá amigos , preciso de uma ajuda na seguinte questão, na verdade a
> resolução porque já tentei muita coisa, já aprendi muita coisa com ela, mas
> mesmo assim não a resolvi.
>
> As três flechas dos três lados (cordas) de um triângulo ABC inscrito em
> uma circunferência de raio R
> valem 1, 2 e 3 calcular a área do triângulo.
>
>
>
> Qualquer ajuda será bem vinda. Obrigado.
>
> Att . Douglas Oliveira
>

-- 
Esta mensagem foi verificada pelo sistema de antiv�rus e
 acredita-se estar livre de perigo.



[obm-l] Problema de geometria.

2016-11-02 Por tôpico Douglas Oliveira de Lima
Olá amigos , preciso de uma ajuda na seguinte questão, na verdade a
resolução porque já tentei muita coisa, já aprendi muita coisa com ela, mas
mesmo assim não a resolvi.

As três flechas dos três lados (cordas) de um triângulo ABC inscrito em uma
circunferência de raio R
valem 1, 2 e 3 calcular a área do triângulo.



Qualquer ajuda será bem vinda. Obrigado.

Att . Douglas Oliveira

-- 
Esta mensagem foi verificada pelo sistema de antiv�rus e
 acredita-se estar livre de perigo.



[obm-l] Teoria dos números

2016-10-14 Por tôpico Douglas Oliveira de Lima
Preciso de uma ajudinha nessa meus caros amigos.

Encontrar todas as soluções inteiras da equação y^2+4=x^3

Douglas Oliveira.

-- 
Esta mensagem foi verificada pelo sistema de antiv�rus e
 acredita-se estar livre de perigo.



[obm-l] Teoria dos números

2016-10-13 Por tôpico Douglas Oliveira de Lima
Olá amigos, preciso de uma ajudinha, não consigo fazer essa:

*Se x e y são inteiros positivos com nenhum fator primo em comum e n é um
quadrado de um inteiro, prove que o número  xn + yn  não é divisível por
(x + y)3.*


*Douglas Oliveira*

-- 
Esta mensagem foi verificada pelo sistema de antiv�rus e
 acredita-se estar livre de perigo.



[obm-l] Re: [obm-l] [obm-l] Questão Geometria

2016-10-04 Por tôpico Douglas Oliveira de Lima
Bom vamos lá, não tem nada de bonito nessa resolução.

Seja O o centro do ex-incirculo de ABC tangente ao lado BC, temos que AO é
bissetriz do ângulo BAC, seja Q a intercessão de AO com BC, e J o pé da
perpendicular tirada de O ao lado AC, sendo BAQ=x, nós teremos CAQ=ACB=x,
AQB=OQC=2x. E OC é bissetriz de BCJ, assim BCO=90-x/2, e sendo P a
intercessao de MO com BC.

1)Aplicando lei dos senos no triângulo AQC teremos

AQ/AC=senx/sen(2x)

2)Agora aplicando no triângulo AMO teremos

AM/MO=sen(QOP)/senx

3)E no triângulo CMO novamente lei dos senos teremos

MC/MO=sen(COP)/cos(x/2)

4)Como AM=MC, dos itens (2) e (3) segue que

sen(QOP)/sen(COP)=senx/cox(x/2)

5) Para o triângulo QPO, nós teremos

sen(QOP)=[(QP)sen(2x)]/PO

6) Para o triângulo CPO, nós teremos

sen(COP)=[(CP).cos(x/2)]/PO

7)Dos itens (5) e (6) podemos concluir que

sen(QOP)/sen(COP)=[(QP).sen(2x)]/[(CP).cos(x/2)]

8)E de (4) e (7) nós temos

senx/cos(x/2)=[(QP).sen(2x)]/[(CP).cos(x/2)], ou melhor QP/CP=senx/sen(2x)

9)Agora de (1) e (8) AQ/AC=QP/CP, donde vem

QAP=CAP e BAP=x+QAP=x+CAP=BPA, ou seja ABP é isosceles e AB=BP.


Um abraço  do
Douglas Oliveira.

Em 1 de outubro de 2016 19:54, vinicius raimundo 
escreveu:

> Será que alguém poria me ajudar na seguinte questão?
>
>
>1.
>
>(Belarus) Seja O o centro do círculo ex-inscrito do triângulo ABC oposto
>ao vértice A. Seja M o ponto médio de AC e seja P a intersec ̧ão das
>retas MO e BC. Prove que se ∠BAC = 2∠ACB, então  AB = BP.
>
>
> --
> Esta mensagem foi verificada pelo sistema de antivírus e
> acredita-se estar livre de perigo.

-- 
Esta mensagem foi verificada pelo sistema de antiv�rus e
 acredita-se estar livre de perigo.



[obm-l] Re: [obm-l] Re: [obm-l] Duas questões de matemática.

2016-08-08 Por tôpico Douglas Oliveira de Lima
OPA, muito obrigado,  mas pensei a respeito de terem um valor inicial.
É como se quando um. Perdesse ele pagaria em que?  Fichas,  como créditos?
Em 08/08/2016 18:41, "Bruno Visnadi" <brunovisnadida...@gmail.com> escreveu:

> Olá
>
> Não sei responder sobre os ângulos suplementares.
> Sobre o problema, não acho que ele esteja mal elaborado.
> O total de dinheiro disputado é 750. Como ambos pagaram e receberam o
> mesmo, cada um pagou e recebeu 375.
> Como 15+20+25+30+35+40+45+50+55+60 = 65+70+75+80+85 = 375, é possível que
> Ricardo tenha ganhado 10 partidas e perdido 5. É impossível que alguém
> tenha ganhado 11, caso contrário a soma mínima seria 440, logo, a maior
> diferença possível é 5, alternativa E.
>
> Em 8 de agosto de 2016 16:45, Douglas Oliveira de Lima <
> profdouglaso.del...@gmail.com> escreveu:
>
>> Olá amigos, gostaria de uma ajuda em uma filosofia e uma questão.
>>
>> 1)Na definição de ângulos suplementares, seria para dois ângulos ou pode
>> ser para mais de dois?
>>
>> 2)(Essa questão gostaria de saber se está mal elaborada) Carlos e
>> Ricardo disputaram 15 partidas de boliche e ao fim de cada partida o
>> perdedor pagava um prêmio em dinheiro para o vencedor. O prêmio para a
>> primeira partida foi R$ 15,00 e o prê- mio de cada partida seguinte foi R$
>> 5,00 a mais do que o valor da partida anterior. Ao final da disputa, ambos
>> receberam o mesmo valor em dinheiro e nenhuma partida terminou empatada.
>> Nes- sas condições, a maior diferença possível entre as vitórias e as
>> derrotas de Ricardo é
>> (A) 4. (B) 3. (C) 7. (D) 6. (E) 5.
>>
>> Att: Douglas Oliveira.
>>
>> --
>> Esta mensagem foi verificada pelo sistema de antivírus e
>> acredita-se estar livre de perigo.
>
>
>
> --
> Esta mensagem foi verificada pelo sistema de antivírus e
> acredita-se estar livre de perigo.

-- 
Esta mensagem foi verificada pelo sistema de antiv�rus e
 acredita-se estar livre de perigo.



[obm-l] Duas questões de matemática.

2016-08-08 Por tôpico Douglas Oliveira de Lima
Olá amigos, gostaria de uma ajuda em uma filosofia e uma questão.

1)Na definição de ângulos suplementares, seria para dois ângulos ou pode
ser para mais de dois?

2)(Essa questão gostaria de saber se está mal elaborada) Carlos e Ricardo
disputaram 15 partidas de boliche e ao fim de cada partida o perdedor
pagava um prêmio em dinheiro para o vencedor. O prêmio para a primeira
partida foi R$ 15,00 e o prê- mio de cada partida seguinte foi R$ 5,00 a
mais do que o valor da partida anterior. Ao final da disputa, ambos
receberam o mesmo valor em dinheiro e nenhuma partida terminou empatada.
Nes- sas condições, a maior diferença possível entre as vitórias e as
derrotas de Ricardo é
(A) 4. (B) 3. (C) 7. (D) 6. (E) 5.

Att: Douglas Oliveira.

-- 
Esta mensagem foi verificada pelo sistema de antiv�rus e
 acredita-se estar livre de perigo.



[obm-l] Re: [obm-l] Re: Equação cotangentes

2016-07-26 Por tôpico Douglas Oliveira de Lima
E o zero?  Não conta?
Em 26/07/2016 00:15, "Israel Meireles Chrisostomo" <
israelmchrisost...@gmail.com> escreveu:

> Opa eu quis dizer (ctg1+i)^n=(ctg1-i)^n com sendo um número complexo
>
> Em 26 de julho de 2016 00:07, Israel Meireles Chrisostomo <
> israelmchrisost...@gmail.com> escreveu:
>
>> como eu posso provar que não existe inteiro n que satisfaça a equação
>>
>> (ctg1+1)^n=(ctg1-1)^n
>> se 1 é dado em radianos, sem usar a transcendência de cotangente de
>> 1?Alguma ideia?
>>
>
>
> --
> Esta mensagem foi verificada pelo sistema de antivírus e
> acredita-se estar livre de perigo.

-- 
Esta mensagem foi verificada pelo sistema de antiv�rus e
 acredita-se estar livre de perigo.



Re: [obm-l] Desigualdade.

2016-06-25 Por tôpico Douglas Oliveira de Lima
Muitissimo obrigado Carlos , era isso mesmo, pois nos livros eles realmente
usam para provar a convexidade.
Em 25 de jun de 2016 23:46, "Carlos Gomes" <cgomes...@gmail.com> escreveu:

> Olá Douglas,
>
> Na maioria dos livros essa desigualdade é usada para definir uma função
> convexa, ou seja, uma função f:[x,y] -->R que satisfaz a condição
> f(ty+(1-t)x)<=tf(y)+(1-t)f(x) com t E [0,1] é, por definição convexa.
>
> No entanto se a função f:[x,y] --> possuir derivada segunda no intervalo
> (x,y), então pode-se mostrar que f(ty+(1-t)x)<=tf(y)+(1-t)f(x) com t E
> [0,1]  se, e somente se f ''(a)>=0, para todo a E [x,y].
>
> Talvez seja isso que vc quer: supondo que f ''(a)>=0, para todo a E
> [x,y], mostrar que f(ty+(1-t)x)<=tf(y)+(1-t)f(x) com t E [0,1]. SE for vc
> faz assim:
>
> Determina a fórmula de Taylor de f em torno de x e de y: sendo
> z=ty+(1-t)x, com t E [0,1] um ponto genérico do intervalo [x,y], segue que
>
> f(x)=f(z)+f'(z)(x-z)+1/2.f ''(c_1)(x-z)^2, com c_1 E [x,z]
> f(y)=f(z)+f'(z)(y-z)+1/2.f ''(c_2)(y-z)^2, com c_2 E [x,z]
>
> multiplicando a primeira por (1-t), a segunda por t a adicionando membro a
> membro, segue que
>
> tf(y)+(1-t)f(x)=f(z)+R  (Faça as contas para conferir!)
>
> onde R:=1/2(1-t).f ''(c_1)(x-z)^2+1/2.t.f ''(c_2)(y-z)^2>=0, pois (1-t), t
> e f '' são >=0.
>
> Assim,
>
> tf(y)+(1-t)f(x)=f(z)+R>=f(z)=f(ty+(1-t)x)
>
> ou seja, f(ty+(1-t)x)<=tf(y)+(1-t)f(x) com t E [0,1].
>
> Abraço, Cgomes.
>
>
>
>
> Em 25 de junho de 2016 20:55, Douglas Oliveira de Lima <
> profdouglaso.del...@gmail.com> escreveu:
>
>> Olá amigos preciso de ajuda na seguinte questão:
>>
>> Mostrar que f(ty+(1-t)x)<=tf(y)+(1-t)f(x) com t E [0,1] e f sendo convexa.
>>
>> Obs: Não usar geometria.
>>
>> Agradeço a ajuda.
>>
>> Douglas Oliveira
>>
>> --
>> Esta mensagem foi verificada pelo sistema de antivírus e
>> acredita-se estar livre de perigo.
>
>
>
> --
> Esta mensagem foi verificada pelo sistema de antivírus e
> acredita-se estar livre de perigo.

-- 
Esta mensagem foi verificada pelo sistema de antiv�rus e
 acredita-se estar livre de perigo.



[obm-l] Desigualdade.

2016-06-25 Por tôpico Douglas Oliveira de Lima
Olá amigos preciso de ajuda na seguinte questão:

Mostrar que f(ty+(1-t)x)<=tf(y)+(1-t)f(x) com t E [0,1] e f sendo convexa.

Obs: Não usar geometria.

Agradeço a ajuda.

Douglas Oliveira

-- 
Esta mensagem foi verificada pelo sistema de antiv�rus e
 acredita-se estar livre de perigo.



Re: [obm-l] Enunciado

2016-04-25 Por tôpico Douglas Oliveira de Lima
2)para esta segunda ligue os centros e aplique lei dos cossenos duas vezes.
Em 25/04/2016 07:04,  escreveu:

>
>
> 1) Um amigo me passou o seguinte enunciado: If
> (x+sqrt(1+x^2).(y+sqrt(1+y^2)=2, find (x+2y).(y+2x). Não está faltando
> informação? Note que x=3/4 e y=0 tornam a equação verdadeira.
>
> 2) Considere 4 circunferências. A maior de diâmetro 3 e as três menores de
> raio 1, 1/2 e r. Determinar r sabendo-se que as três menores são tangentes
> internamente com a maior e tangentes entre si ( ou seja todas tangentes a
> todas). Provavelmente esse exercício já deve ter sido feito por aqui. Quem
> puder ajudar, agradeço antecipadamente. Obrigado.
>
>
>   Roy.
>
>
> --
> Esta mensagem foi verificada pelo sistema de antivírus e
> acredita-se estar livre de perigo.
>

-- 
Esta mensagem foi verificada pelo sistema de antiv�rus e
 acredita-se estar livre de perigo.



[obm-l] Preciso de uma ajuda.

2016-02-29 Por tôpico Douglas Oliveira de Lima
Olá caros amigos, encontrei uma resolução que diz o seguinte:

"Queremos demonstrar que, no plano orientado, (DM,DM') + (D'M,D'M')=2k,
sendo escolhido, como unidade, o ângulo reto e sendo h um inteiro
algébrico."

Não entendi muito bem a linguagem a respeito de plano orientado e inteiro
algébrico, gostaria de uma ajuda(esclarecimento a respeito do assunto).
Desde já agradeço a ajuda.


Douglas Oliveira de Lima.

-- 
Esta mensagem foi verificada pelo sistema de antiv�rus e
 acredita-se estar livre de perigo.



Re: [obm-l] Ajuda numa desigualdade.

2016-01-28 Por tôpico Douglas Oliveira de Lima
Opa Marcelo, muito obrigado mesmo, eu estou procurando uma solução daquelas
tipo
desigualdades, onde efetuamos uma estratégia para chegar no resultado, tipo
uma daquelas que tu encontra no livro de combinatória do MOrgado(o problema
das apostas).
Mas valeu, se conseguir uma dessas me manda novamente por favor.
Abraço
Douglas Oliveira

Em 28 de janeiro de 2016 01:26, Marcelo Salhab Brogliato <msbro...@gmail.com
> escreveu:

> Oi, Douglas, tudo bem?
>
> Se provarmos que f(x) = (1 + 1/x)^x é estritamente crescente, então está
> provada sua desigualdade.
>
> Uma maneira é fazer isso usando cálculo. Seja g(x) = ln(f(x)) = x ln(1 +
> 1/x). Assim, se provarmos que g(x) é estritamente crescente, então f(x)
> também será (exercício: prove essa afirmação).
>
> g'(x) = ln(1 + 1/x) + x * (-1/x^2) / (1 + 1/x)  = ln(1 + 1/x) - (1/x) / (1
> + 1/x) = ln(1 + 1/x) - 1/(1+x)
>
> Temos que mostrar que g'(x) > 0 para todo x.
>
> Sabemos que ln(x) < x - 1, para x != 1. Aplicando essa desigualdade em
> 1/x, temos: ln(1/x) < 1/x - 1 => ln(x) > 1 - 1/x, para x != 1.
>
> Aplicando a desigualdade acima em 1+1/x, temos: ln(1+1/x) > 1 - 1/(1 +
> 1/x) = (1/x) / (1 + 1/x) = 1/(1+x). Logo: ln(1+1/x) > 1/(1+x) => g'(x) > 0
> para todo x (já que 1+1/x > 1).
>
> Abraços,
> Salhab
>
> 2016-01-28 0:34 GMT-02:00 Douglas Oliveira de Lima <
> profdouglaso.del...@gmail.com>:
>
>> Olá caros amigos, gostaria de uma ajuda na seguinte desigualdade
>> (1+1/n)^n<(1+1/n+1)^(n+1), para n natural.
>>
>> Agradeço desde já.
>>
>>
>


RE: [obm-l] Ajuda numa desigualdade.

2016-01-28 Por tôpico Douglas Oliveira de Lima
Erro? Bom no meu celular acho que saiu as fórmulas todas fora de ordem rs
Em 28/01/2016 16:02, "Bruno Lira" <brunotorne...@hotmail.com> escreveu:

> Primeiramente, tome a função logaritmântica f(x) = ln(x) cujo
>
> domínio é o conjuntos dos números reais maiores que ou
>
> igual a zero. Note que a função f é injetora. Portanto,
>
> para provarmos que:
>
>
> n n+1
>
> ( 1 + *1* ) < ( 1 + * 1 * )
>
> (   n ) ( n+1 )
>
>
> basta provar que:
>
>
>(n)   (  n+1)
>
> ln( ( 1 + *1* ) ) < ln( ( 1 + *  1 *)  )
>
>( (   n ) )   ( ( n+1)  ) .
>
>
> De fato, temos que:
>
>
>(n)( n+1)
>
> ln( ( 1 + *1* )  ) – ln( ( 1 +*   1   *)   ) =
>
>( (  n )  )((  n + 1)   )
>
>
>(n)(n+1)
>
> ln( ( *n** + 1 *)  ) – ln( ( *n** + **2* ) ) =
>
>( (n) )( ( n + 1 ))
>
>
>(   2n  )
>
> ln( (*  n + 1  *) . *n+1* ) ; Das propriedades de logaritmo.
>
>( (n (n+2))   n+2 )
>
>
> Daí:
>
>
>( n   )
>
> ln( ( *n^2 + 2**n** + 1 *) . *n+1* )
>
>( (n^2 + 2n   )n+2)
>
>
> Como n^2 + 2n < n^2 + 2n + 1 e n+1 < n + 2 temos que:
>
>
> n
>
> ( *n^2 + 2**n** + 1 *) . *n+1*<1
>
> (n^2 + 2n)   n+2
>
>
> E da injetividade da função f temos:
>
>
>( n   )
>
> ln( ( *n^2 + 2**n** + 1 *) . *n+1* )   <ln(1)=0
>
>( (n^2 + 2n   )n+2)
>
>
> Isto é:
>
>
>(n)(n+1)
>
> ln( ( 1 + *1* )  ) – ln( ( 1 + *1 *)  )<0
>
>( (  n )  )( ( n+1  )  )
>
>
> Logo,
>
>
> n n+1
>
> ( 1 + *1* ) < ( 1 + *1 *)
>
> (   n ) ( n+1 )
>
>
> C.Q.D
>
> P.S.: Se tiver algum erro me avisem por favor.
> --
> From: esdrasmunizm...@gmail.com
> Date: Thu, 28 Jan 2016 12:18:03 -0300
> Subject: Re: [obm-l] Ajuda numa desigualdade.
> To: obm-l@mat.puc-rio.br
>
> L = ((1+1/(n+1))^(n+1))/(1+1/n)^n = ((1 - 1/(n+1)²)^n)((n+2)/(n+1))
>
> Use que (1 - x)^n > 1 - nx, Para x \in (0, 1)
>
> L > (1 - n/(n+1)²)((n+2)/(n+1)) = ((n²+n+1)/(n²+2n+1))((n+2)/(n+1))
> = (n³+3n²+3n+2)/(n³+3n²+3n+1) > 1.
>
>
>
> Esse último termo é maior que 1.
>
> Em 28 de janeiro de 2016 09:41, Douglas Oliveira de Lima <
> profdouglaso.del...@gmail.com> escreveu:
>
> Opa Marcelo, muito obrigado mesmo, eu estou procurando uma solução
> daquelas tipo
> desigualdades, onde efetuamos uma estratégia para chegar no resultado,
> tipo uma daquelas que tu encontra no livro de combinatória do MOrgado(o
> problema das apostas).
> Mas valeu, se conseguir uma dessas me manda novamente por favor.
> Abraço
> Douglas Oliveira
>
> Em 28 de janeiro de 2016 01:26, Marcelo Salhab Brogliato <
> msbro...@gmail.com> escreveu:
>
> Oi, Douglas, tudo bem?
>
> Se provarmos que f(x) = (1 + 1/x)^x é estritamente crescente, então está
> provada sua desigualdade.
>
> Uma maneira é fazer isso usando cálculo. Seja g(x) = ln(f(x)) = x ln(1 +
> 1/x). Assim, se provarmos que g(x) é estritamente crescente, então f(x)
> também será (exercício: prove essa afirmação).
>
> g'(x) = ln(1 + 1/x) + x * (-1/x^2) / (1 + 1/x)  = ln(1 + 1/x) - (1/x) / (1
> + 1/x) = ln(1 + 1/x) - 1/(1+x)
>
> Temos que mostrar que g'(x) > 0 para todo x.
>
> Sabemos que ln(x) < x - 1, para x != 1. Aplicando essa desigualdade em
> 1/x, temos: ln(1/x) < 1/x - 1 => ln(x) > 1 - 1/x, para x != 1.
>
> Aplicando a desigualdade acima em 1+1/x, temos: ln(1+1/x) > 1 - 1/(1 +
> 1/x) = (1/x) / (1 + 1/x) = 1/(1+x). Logo: ln(1+1/x) > 1/(1+x) => g'(x) > 0
> para todo x (já que 1+1/x > 1).
>
> Abraços,
> Salhab
>
> 2016-01-28 0:34 GMT-02:00 Douglas Oliveira de Lima <
> profdouglaso.del...@gmail.com>:
>
> Olá caros amigos, gostaria de uma ajuda na seguinte desigualdade
> (1+1/n)^n<(1+1/n+1)^(n+1), para n natural.
>
> Agradeço desde já.
>
>
>
>
>
>
> --
> Esdras Muniz Mota
> Mestrando em Matemática
> Universidade Federal do Ceará
>
>
>


[obm-l] Ajuda numa desigualdade.

2016-01-27 Por tôpico Douglas Oliveira de Lima
Olá caros amigos, gostaria de uma ajuda na seguinte desigualdade
(1+1/n)^n<(1+1/n+1)^(n+1), para n natural.

Agradeço desde já.


Re: [obm-l] Livros

2016-01-14 Por tôpico Douglas Oliveira de Lima
Tambem gostaria se pudesse please.
profdouglaso.del...@gmail.com
Em 14/01/2016 11:05, "Jefferson Cândido"  escreveu:

> Muito bom! Se puder mandar também para meu e-mail, jjjeffer...@gmail.com,
> agradeço!
>
> Em 13 de janeiro de 2016 21:45, Vanderlei Nemitz 
> escreveu:
>
>> *PROBLEMAS DE ALTA DIFICULDAD - 300 Problemas Resolvidos*
>> *Métodos de Resoluções e Demonstrações de Desigualdades - ** 367
>> Problemas*
>> *Métodos Alternativos para a Resolução de Equações e Inequações - 350
>> Problemas Resolvidos*
>>
>> *Qualquer um desses já seria uma grande ajuda!*
>>
>> *Obrigado!*
>>
>> Em 13 de janeiro de 2016 21:33, regis barros 
>> escreveu:
>>
>>> Olá Vanderlei
>>> Quais livros do suprun você precisa?
>>>
>>> Regis
>>>
>>>
>>> Em Quarta-feira, 13 de Janeiro de 2016 14:35, Vanderlei Nemitz <
>>> vanderma...@gmail.com> escreveu:
>>>
>>>
>>> Boa tarde! Alguém tem os PDFs dos livros do Suprún? Pode ser até em
>>> russo mesmo! Ou mesmo tenha e queira vender os livros físicos? Preciso
>>> muito deles, mas está em falta.
>>>
>>> Obrigado!
>>>
>>>
>>>
>>
>
>
> --
> É preciso amar as pessoas como se não houvesse amanhã...
>
> Jefferson Cândido -
>


[obm-l] Uma boa questão de tetraedro.

2015-12-12 Por tôpico Douglas Oliveira de Lima
Olá amigos vou postar aqui duas questões, a primeira é proposta e a segunda
estou na dúvida precisando de ajuda.

1)Mostre que todo tetraedro que tem seus quatro ângulos triedros iguais,
tem suas arestas opostas iguais.

2)Se a soma dos três ângulos que figuram em cada um dos quatro triedros  de
um tetraedro vale 180 graus, então esse tetraedro é isósceles, ou seja,
possui arestas opostas iguais.


Abraço do Douglas Oliveira.

-- 
Esta mensagem foi verificada pelo sistema de antiv�rus e
 acredita-se estar livre de perigo.



[obm-l] Re: [obm-l] Quadriláteros Inscritíveis

2015-11-05 Por tôpico Douglas Oliveira de Lima
Note que os triangulos ABO e ACO sao senelhantes, logo pelas proporcoes dos
lados vai perceber que ADO e ECO sao semelhantes e pronto.

Abraco
Douglas Oliveira
Em 02/11/2015 00:22, "Lucas Melo"  escreveu:

> Alguém poderia resolver essa questão?
> (São Petersburgo 1996) Seja ABC um triângulo tal que BÂC=60º. Seja também
> O um ponto no interior de ABC para o qual AÔB=BÔC = 120º . Se D, E são os
> pontos médios dos lados AB, AC, prove que A, D, E, O são concíclicos.
>
> P.s. Eu poderia dizer que esse triângulo é equilátero? Se sim, como
> demonstrar?
> --
> Esta mensagem foi verificada pelo sistema de antivírus e
>  acredita-se estar livre de perigo.
>
>
> =
> Instru�ões para entrar na lista, sair da lista e usar a lista em
> http://www.mat.puc-rio.br/~obmlistas/obm-l.html
> =
>

-- 
Esta mensagem foi verificada pelo sistema de antiv�rus e
 acredita-se estar livre de perigo.



[obm-l] Conicas

2015-10-29 Por tôpico Douglas Oliveira de Lima
Olá caros amigos, gostaria de uma ajuda no seguinte problema:

PROBLEMA: Encontrar a abscissa da parábola de equação
 x^2+2xy+y^2-2x+4y+1=0.

OBS: Essa questão caiu na prova do ITA acho que de 2012, e vi uma solução
que envolvia limites do qual não compreendi muito bem.
Sei portanto como usar a rotação de eixos e também através de
diagonalização. Mas gostaria de saber se existe outro modo de chegar a tal
abscissa.

Desde já obrigado.
Forte abraço do Douglas Oliveira.

-- 
Esta mensagem foi verificada pelo sistema de antiv�rus e
 acredita-se estar livre de perigo.



Re: [obm-l] Geometria

2015-09-08 Por tôpico Douglas Oliveira de Lima
É BC ou DC?
Em 08/09/2015 10:17, "Pedro José"  escreveu:

> Bom dia!
>
> Uma ajuda.
>
> Seja um triângulo ABC, são traçadas três cevianas que se interceptam em um
> único ponto D, no interior do triângulo.
> Sejam M, N e P os pés das cevianas e DM, DN e DP são congruentes com
> medida igual a 3.
> A soma das medidas dos segmentos DA, DB e BC é igual a 143. Calcule o
> produto das medidas desses segmentos.
>
> Grato,
> PJMS
>
> --
> Esta mensagem foi verificada pelo sistema de antivírus e
> acredita-se estar livre de perigo.

-- 
Esta mensagem foi verificada pelo sistema de antiv�rus e
 acredita-se estar livre de perigo.



[obm-l] Re: [obm-l] Re: [obm-l] Re: [obm-l] Re: [obm-l] Problema muito bacana de teoria dos números

2015-08-08 Por tôpico Douglas Oliveira de Lima
Bom , vamos lá:

1)Como N possui 12 divisores, temos que 1 será o menor e N será o maior.

2)Usando uma propriedade bem conhecida teremos dk.d(13-k)=t, ou seja o
divisor de indice k e o de índice 13-k.

3)Como o divisor de índice d4-1 é igual a (d1+d2+d4)d8, teremos que
d1+d2+d4 é divisor também logo, da.d8 é igual ao divisor de índice d4-1, e
qualquer divisor será menor ou igual a N, assim devemos ter a menor ou
igual a 5.

4)Fazendo d1+d2+d4=d5 , que o divisor de índice d4-1 será igual a
d5.d8=N=d12, logo d4-1=12, d4=13.

5)Agora como d1=1 e d4=5 temos as possibilidades para d2 (2,3,5,7,11).

6)Um número ccom 12 divisores possui 1, 2 ou 3 primos, pois (2^2).3

7)Se d2=2, d5=16 e isso é absurdo pois 4 e 8 também seriam divisores.

8)Assim se d2=3 teremos d5=17.

9) Assim como N possui 12 divisores com fatores 3, 13, 17 temos algumas
possibilidades para N, que seriam N=(3^x).(13^y).(17^z) com (x,y,z)=(2,1,1)
ou (1,2,1) ou (1,1,2).

10)Assim das três apenas uma vale que é (x,y,z)=(2,1,1)



Abraços do Douglas Oliveira

Em 7 de agosto de 2015 14:58, Pedro José petroc...@gmail.com escreveu:

 Boa tarde!

 Saulo,

 Se 2 e 3 são divisores 6 também será.

 Achei esse problema casca grossa.

 Saudações,
 PJMS

 Em 6 de agosto de 2015 23:25, Mauricio de Araujo 
 mauricio.de.ara...@gmail.com escreveu:

 N = 1989.

 Em 6 de agosto de 2015 14:50, saulo nilson saulo.nil...@gmail.com
 escreveu:

 d4-1=11
 d4=12
 d1=1
 d2=2
 d3=
 d11=(1+2+12)d8=15*17=255
 1,2,3,12,13,14,15,17,18,19,255, produto deles.

 2015-08-06 13:14 GMT-03:00 Mauricio de Araujo 
 mauricio.de.ara...@gmail.com:

 Um número natural N tem exatamente 12 divisores (incluindo 1 e N), tais
 que, colocados em ordem crescente temos d1  d2  d3  ...  d12.
 Sabe-se que o divisor que possui o índice d4 - 1 é igual ao produto (d1
 + d2 + d4).d8. Achar N.

 --
 Abraços

 oɾnɐɹɐ ǝp oıɔıɹnɐɯ


 --
 Esta mensagem foi verificada pelo sistema de antivírus e
 acredita-se estar livre de perigo.



 --
 Esta mensagem foi verificada pelo sistema de antivírus e
 acredita-se estar livre de perigo.




 --
 Abraços

 oɾnɐɹɐ ǝp oıɔıɹnɐɯ


 --
 Esta mensagem foi verificada pelo sistema de antivírus e
 acredita-se estar livre de perigo.



 --
 Esta mensagem foi verificada pelo sistema de antivírus e
 acredita-se estar livre de perigo.


-- 
Esta mensagem foi verificada pelo sistema de antiv�rus e
 acredita-se estar livre de perigo.



[obm-l] Problema

2015-06-22 Por tôpico Douglas Oliveira de Lima
Olá caros colegas, gostaria de uma ajuda no seguinte problema:

Em uma reta há 1999 bolinhas. Algumas são verdes e as demais azuis(poderiam
ser todas verdes ou todas azuis). Debaixo de cada bolinha escrevemos o
número igual a soma da quantidade de bolinhas verdes à sua direita dela
mais a quantidade de bolinhas azuis a esquerda dela. Se, na sequência de
números assim obtida, houver exatamente três números que aparecem uma
quantidade ímpar de vezes, quais podem ser estes números?


Abraço


Douglas Oliveira

-- 
Esta mensagem foi verificada pelo sistema de antiv�rus e
 acredita-se estar livre de perigo.



[obm-l] Quadrados numa malha 10x10

2015-06-15 Por tôpico Douglas Oliveira de Lima
Olá, caros amigos, preciso de uma ajuda no seguinte problema:
Quantos quadrados podemos formar numa malha 10x10?

Obs: Se souberem de algum artigo ou algum material escrito falando sobre o
assunto, ate mesmo esses livros de puzzles voltados para a matemática e
puderem me indicar , agradeço desde já.

Um abraço do Douglas Oliveira

-- 
Esta mensagem foi verificada pelo sistema de antiv�rus e
 acredita-se estar livre de perigo.



Re: [obm-l] Problema de Desigualdade

2015-06-11 Por tôpico Douglas Oliveira de Lima
Então não é trabalhoso, mas (a/b)^2 = 1 + a/b - b/a não deveria ser
provado?
Desenvolvendo da pra ver que é, neste caso tem mais conta pra fazer.

Forte abraço
Douglas Oliveira.



Em 10 de junho de 2015 12:00, Alexandre Antunes 
prof.alexandreantu...@gmail.com escreveu:


 Bom dia,

 Estou no trabalho, mas vou arriscar a minha primeira resposta no grupo.

 Desenvolvi os dois lados da expressao.

 (a/b)^2 + (b/c)^2 + (c/a)^2 = 3 + (a/b + b/c + c/a) - (b/a + c/b + a/c)

 Como (a/b)^2 = 1 + a/b - b/a
 O mesmo para os demais termos

 Fica provado a proposição.

 O que acham desse trabalhoso caminho?!?!
 Em 10/06/2015 09:00, Pacini Bores pacini.bo...@globo.com escreveu:

 Ok Mariana.

 Abraços

 Pacini

 Em 9 de junho de 2015 21:11, Mariana Groff 
 bigolingroff.mari...@gmail.com escreveu:

 Oi Pacini,
 Fiz do seguinte modo:
 f (x)=x^2-x+1/x=1 = x^3-x^2+1=x = x^3-x^2-x+1=0 =x^2
 (x-1)-(x-1)=0 = (x^2-1)(x-1)=0
 O que podemos ver que é verdade, analisando ambos os casos: em que x=1
 e o caso em que 0 x 1.
 Abraços,
 Mariana
  Em 09/06/2015 20:55, Pacini Bores pacini.bo...@globo.com escreveu:

 Oi Mariana,

 Determinei o mínimo da função usando a derivada. Não entendi o seu
 caminho, pois a função é

 f(x) = x^2-x+1/x.

 Abraços

 Pacini

 Em 9 de junho de 2015 18:09, Mariana Groff 
 bigolingroff.mari...@gmail.com escreveu:

Oi Pacini,
 Compreendi seu raciocínio. Para provar que f(x)=1, basta analisarmos
 que (x^2-1)(x-1)=0, o que verifica-se pois se x=1, o produto é 
 claramente
 não-negativo e se 0x1, vemos que, tanto x^2-1 quanto x-1 são negativos,
 tornando o produto positivo, isso?


 Em 9 de junho de 2015 11:48, Pacini Bores pacini.bo...@globo.com
 escreveu:

 Oi Mariana,
  Observe que provar  a desigualdade pedida  é equivalente  provar que
 :

 {(a/b)^2-a/b+b/a} + {(b/c)^2-b/c+c/b} +{(c/a)^2-c/a+a/c} =3, ok ?

 Agora façamos o seguinte :

 Seja f(x)= x^2-x+1/x, verifique que para x0 o valor mínimo de f é 1.

 Donde teremos a desigualdade provada.

  Estou certo pessoal ?

 Abraços

 Pacini


 Em 8 de junho de 2015 20:30, Raphael Aureliano raphael0...@gmail.com
  escreveu:

 Ah não, desculpa, errei em Cauchy ...

 Att.
 Raphael
 Em 08/06/2015 20:27, Raphael Aureliano raphael0...@gmail.com
 escreveu:

 MA=MG
 LE=(a/b+b/c+c/a)^2=(3cbrt(abc/abc))^2 =9

 Por Cauchy
 LD=(a+b+c)(1/a+1/b+1/c)=(sqrt(a/a) +sqrt(b/b)+sqrt(c/c))^2 =9

 LE=9=LD
  Em 08/06/2015 19:20, Mariana Groff 
 bigolingroff.mari...@gmail.com escreveu:

 Boa Noite,

 (British Mathematical Olympiad - Round 2 - 2005)
 Sejam a,b e c reais positivos.
 Prove que

 (a/b+b/c+c/a)^2=(a+b+c)(1/a+1/b+1/c)

 Atenciosamente,
 Mariana

 --
 Esta mensagem foi verificada pelo sistema de antivírus e
 acredita-se estar livre de perigo.


 --
 Esta mensagem foi verificada pelo sistema de antivírus e
 acredita-se estar livre de perigo.



 --
 Esta mensagem foi verificada pelo sistema de antivírus e
 acredita-se estar livre de perigo.



 --
 Esta mensagem foi verificada pelo sistema de antivírus e
 acredita-se estar livre de perigo.



 --
 Esta mensagem foi verificada pelo sistema de antivírus e
 acredita-se estar livre de perigo.


 --
 Esta mensagem foi verificada pelo sistema de antivírus e
 acredita-se estar livre de perigo.



 --
 Esta mensagem foi verificada pelo sistema de antivírus e
 acredita-se estar livre de perigo.


 --
 Esta mensagem foi verificada pelo sistema de antivírus e
 acredita-se estar livre de perigo.


-- 
Esta mensagem foi verificada pelo sistema de antiv�rus e
 acredita-se estar livre de perigo.



Re: [obm-l] Contagem

2015-06-01 Por tôpico Douglas Oliveira de Lima
Escolha 8 em 30, isso nos dará 30!/8!22!

Douglas Oliveira.

Em 26 de maio de 2015 22:51, marcone augusto araújo borges 
marconeborge...@hotmail.com escreveu:

 De quantas maneiras é possível cortar um colar(na forma de círculo
 inteiro) de 30
 pérolas em 8 partes(só é permitido cortar entre as pérolas)?

 --
 Esta mensagem foi verificada pelo sistema de antivírus e
 acredita-se estar livre de perigo.


-- 
Esta mensagem foi verificada pelo sistema de antiv�rus e
 acredita-se estar livre de perigo.



Re: [obm-l] inteiros positivos

2015-05-27 Por tôpico Douglas Oliveira de Lima
Pense que x só pode assumir 4 formas,  4k, 4k+1, 4k+2, 4k+3.
Em 27/05/2015 10:05, Pedro José petroc...@gmail.com escreveu:

 Bom dia!

 7^3 ≡ 4*7 ≡ 1 (mod9) e não 7^3 ≡ 3*7 ≡ 1 (mod9)

 Em 27 de maio de 2015 09:52, Pedro José petroc...@gmail.com escreveu:

 Bom dia!

 Douglas,
 há valores ímpares de x que atendem 7^x≡ 4 (mod9)

 7^2 ≡ 4 (mod9) == x ≡  2 (mod3)

 7^1 ≡ 7 (mod9)
 7^2 ≡4 (mod9)
 7^3 ≡ 3*7 ≡ 1 (mod9)
 == 7^(2+3k) ≡ 7^2*(7^3)^k ≡ 4 (mod9)


 -- Mensagem encaminhada --
 De: Douglas Oliveira de Lima profdouglaso.del...@gmail.com
 Data: 26 de maio de 2015 23:37
 Assunto: Re: [obm-l] inteiros positivos
 Para: obm-l@mat.puc-rio.br obm-l@mat.puc-rio.br



 Bom, é fácil ver que x=1 e y=1 satisfaz a equação, assim caso y seja
 maior ou igual a 2,
 teremos que 7^x=4 (mod 9), desta forma x=2 (mod 4), ou podemos dizer que
 x é par da forma 2k,
 logo 7^2k-3^y=4, (7^k+2)(7^k-2)=3^y, mas nao existem duas potências de 3
 cuja diferença vale 4.
 Assim só existe uma solução.

 Abraço.
 Douglas Oliveira

 Em 26 de maio de 2015 22:41, marcone augusto araújo borges 
 marconeborge...@hotmail.com escreveu:

 Determine todos os inteiros positivos x e y tais que 7^x - 3^y = 4

 --
 Esta mensagem foi verificada pelo sistema de antivírus e
 acredita-se estar livre de perigo.



 --
 Esta mensagem foi verificada pelo sistema de antivírus e
 acredita-se estar livre de perigo.



 --
 Esta mensagem foi verificada pelo sistema de antivírus e
 acredita-se estar livre de perigo.

-- 
Esta mensagem foi verificada pelo sistema de antiv�rus e
 acredita-se estar livre de perigo.



Re: [obm-l] inteiros positivos

2015-05-26 Por tôpico Douglas Oliveira de Lima
Bom, é fácil ver que x=1 e y=1 satisfaz a equação, assim caso y seja maior
ou igual a 2,
teremos que 7^x=4 (mod 9), desta forma x=2 (mod 4), ou podemos dizer que x
é par da forma 2k,
logo 7^2k-3^y=4, (7^k+2)(7^k-2)=3^y, mas nao existem duas potências de 3
cuja diferença vale 4.
Assim só existe uma solução.

Abraço.
Douglas Oliveira

Em 26 de maio de 2015 22:41, marcone augusto araújo borges 
marconeborge...@hotmail.com escreveu:

 Determine todos os inteiros positivos x e y tais que 7^x - 3^y = 4

 --
 Esta mensagem foi verificada pelo sistema de antivírus e
 acredita-se estar livre de perigo.


-- 
Esta mensagem foi verificada pelo sistema de antiv�rus e
 acredita-se estar livre de perigo.



Re: [obm-l] Diofantina

2015-05-20 Por tôpico Douglas Oliveira de Lima
*Hummm acho que consegui!*



*Vamos lá, fiz z^2=y^4(assim z deve ser um quadrado perfeito). Logo a
equação x^2-2z^2=1. Essa equação representa uma hipérbole, então dá pra
encontrar as soluções racionais nessa curva, fácil ver que (x,y)=(1,0) é
solução, assim considere uma reta que passa por esse ponto e intercepta a
hip. no outro ponto (a,b), logo a equação dessa reta será y-0=m(x-1), ou
y=m(x-1).*

*Agora substituindo na equação da hip. Teremos a seguinte equação do
segundo grau (1-m^2)x^2+4m^2x-2m^2-1=0, onde conhecemos uma das raízes que
é “1”.*

*Agora podemos usar o produto para encontrar a outra “a” , assim *

*1.a=(-2m^2-1)/(1-2m^2), logo temos todas as soluções racionais da curva *

*x^2-2z^2=1, que serão (x,z)=( (-2m^2-1)/(1-2m^2),m((-2m^2-1)/(1-2m^2) -1)
).*

*Agora basta verificar para quais valores de m x=(-2m^2-1)/(1-2m^2) será
inteiro, podemos escrever x=1- 2/(1-2m^2), assim 1-2m^2 deve ser divisor de
2, ou seja, 1-2m^2=1, 1-2m^2=-1, 1-2m^2=2(não real), 1-2m^2=-2(não
inteira), onde temos respectivamente os valores m=0, m=-1, m=1 , assim os
valores de x só podem ser -1 ou 3, e os de z só podem ser 0,2,-2, mas como
z deve ser um quadrado perfeito só pode ser igual a 0, assim (x,y)=(1,0),
ou (x,y)=(-1,0).*



*Pronto acabou.*



*Valeu, um abraço*

*Douglas Oliveira.*


Em 20 de maio de 2015 12:28, Marcelo Salhab Brogliato msbro...@gmail.com
escreveu:

 x^2 - 2y^4 = 1
 x^2 - 1 = 2y^4
 (x+1)(x-1) = 2y^4

 Como 2y^4 é par, x tem que ser ímpar. Assim, x = 2k + 1.
 Substituindo:
 (2k+2)(2k) = 2y^4
 4k(k+1) = 2y^4
 2k(k+1) = y^4

 Como 2k(k+1) é par, y tem que ser par. Assim, y = 2u
 Substituindo:
 2k(k+1) = 16u^4
 k(k+1) = 8u^4

 Como k e k+1 tem paridades opostas, temos que 8|k ou 8|k+1.

 Caso 1:
 Se 8|k, então: k = 8w, logo 8w(8w+1) = 8u^4 = u^4 = w(8w+1)
 Claramente, u = w = 0 é solução. Nesse caso, k = 0, logo, x = 1 e y = 0.
 Analisando essa equação módulo 2, temos u == w (mod2).
 Acho que não tem outra solução, mas não consegui achar uma contradição.
 Testei e não tem solução de w = 1 até 1 milhão.

 Caso 2:
 Se 8|k+1, então: k+1 = 8w, logo (8w-1)8w = 8u^4 = u^4 = w(8w-1)
 Novamente, u = w = 0 é solução. Nesse caso, k = -1, logo, x = -1 e y = 0.
 Analisando essa equação módulo 2, temos u == w (mod2).
 Acho que não tem outra solução, mas não consegui achar uma contradição.
 Testei e não tem solução de w = 1 até 1 milhão.

 Acho que as únicas soluções são (1, 0) e (-1, 0).

 Estou sem tempo agora, mas posso tentar mais tarde.

 Abraços,
 Salhab



 2015-05-15 14:24 GMT-03:00 Douglas Oliveira de Lima 
 profdouglaso.del...@gmail.com:

 Encontrar todas as soluções inteiras de x^2-2y^4=1.

 Douglas Oliveira

 --
 Esta mensagem foi verificada pelo sistema de antivírus e
 acredita-se estar livre de perigo.



 --
 Esta mensagem foi verificada pelo sistema de antivírus e
 acredita-se estar livre de perigo.


-- 
Esta mensagem foi verificada pelo sistema de antiv�rus e
 acredita-se estar livre de perigo.



Re: [obm-l] Diofantina

2015-05-20 Por tôpico Douglas Oliveira de Lima
Não encontrei soluções inteiras pra ela além da (1,0)

Douglas Oliveira.
Em 20/05/2015 10:27, Pedro José petroc...@gmail.com escreveu:

 Boa noite!

 x^2 - 1 = 2 y^4 == (x+1) (x-1) = 2 y^4 .
 Como x Ɛ 2Z +1 == (x+1) e (x-1)  Ɛ 2Z  == y  Ɛ 2Z == y = 2k, k  Ɛ Z





 Em 15 de maio de 2015 14:24, Douglas Oliveira de Lima 
 profdouglaso.del...@gmail.com escreveu:

 Encontrar todas as soluções inteiras de x^2-2y^4=1.

 Douglas Oliveira

 --
 Esta mensagem foi verificada pelo sistema de antivírus e
 acredita-se estar livre de perigo.



 --
 Esta mensagem foi verificada pelo sistema de antivírus e
 acredita-se estar livre de perigo.

-- 
Esta mensagem foi verificada pelo sistema de antiv�rus e
 acredita-se estar livre de perigo.



  1   2   3   >